MS II Q3

Pataasin ang iyong marka sa homework at exams ngayon gamit ang Quizwiz!

A nurse is collecting data on a client's cardiac functioning and auscultates an S3 sound. The nurse is aware that this sound represents which of the following/? 1. atrial gallop 2. ventricular gallop 3. closure of the aortic valve 4. closure of the pulmonic valve

Ventricular gallop

A nurse is caring for a client who is on Warfarin therapy for atrial fibrillation. The client INR is 5.2. Which of the following medication should the nurse prepare to administer? A. Vitamin K B. Protamine sulfate C. Atropine D. Epinephrine

Vitamin K

A nurse is teaching a client who has heart failure about the need to limit sodium in the diet to 2,000 mg daily. Which of the following foods should be consumed in limited quantities? (Select all that apply.) A. Cheddar cheese, 2 oz B. Hot dog C. Canned tuna, 3 oz D. Roast chicken breast, 3 oz E. Baked ham, 3 oz

abce

A nurse is teaching a client who has a new prescription for colesevelam to lower his lower density lipoprotein level. which of the following instructions should the nurse include? A. "Take this medication 4 hr after other medications." B. "Reduce fluid intake." C. "Take this medication on an empty stomach." D. "Chew tablets before swallowing."

take this medicafion 4 hr after other medications

A nurse is reinforcing teaching with a client who has heart failure about the need to limit sodium in the diet to 2,000 mg daily. Which of the following foods should the nurse recommend for the client? select all that apply A. 1 slice cheddar cheese B. 1 medium beef hot dog C. 3 oz. Atlantic salmon D. 3 oz. roasted chicken breast E. 2 oz. lean baked ham

A. 1 slice of cheddar cheese contains 180 mg Na+ C. 3 oz Atlantic salmon contains 37 mg Na+ D. 3 ox roasted chicken breast contains 62 mg Na+

A nurse on a telemetry unit is caring for a client who has unstable angina and is reporting chest pain with a severity of 6 on a 0-10 scale. The nurse administers 1 sublingual nitroglycerin tablet. After 5 min, the client states that his chest pain is now a severity of 2. Which of the following actions should the nurse take? a. Admin another nitroglycerin tablet b. Initiate a peripheral IV c. Call the Rapid Response Team d. Obtain an ECG

A. Admin. Another nitroglycerin tablet--- Admin guideline for sublingual nitroglycerin indicate that it is appropriate to admin another tablet 5 min after the first one if the client is still reporting pain

A nurse is teaching a client who has a new prescription for clopidogrel. Which of the following instructions should the nurse include in the teaching? (Select all that apply.) A. Avoid the consuming grapefruit while taking this medication. B. Monitor for the presence of black, tarry stools. C. Use an electric razor when shaving. D. Schedule a weekly PT test. E. Limit food sources containing vitamin K while taking this medication.

A. Avoid the consuming grapefruit while taking this medication. B. Monitor for the presence of black, tarry stools

A nurse is caring for a client in a clinic who asks the nurse why her provider prescribed 1 aspirin per day. Which of the following is an appropriate response by the nurse? A. "Aspirin reduces the formation of blood clots that could cause a heart attack." B. "Aspirin relieves the pain due to myocardial ischemia." C. "Aspirin dissolves clots that are forming in your coronary arteries." D. "Aspirin relieves headaches that are caused by other medications."

A. "Aspirin reduces the formation of blood clots that could cause a heart attack."

A nurse is caring for a client who is undergoing conservative treatment for deep-vein thrombosis. The client asks the nurse what will happen to the clot. Which of the following responses should the nurse make? A. "Your body has a process called fibrinolysis that will eventually dissolve the clot." B. "Your body has a mechanism that will keep the clot stable in its present location." C. "The clot will break into tiny fragments and float harmlessly in your bloodstream." D. "Treatment with heparin will dissolve the clot and keep other clots from forming."

A. "Your body has a process called fibrinolysis that will eventually dissolve the clot."--- Fibrinolysis is a process that breaks down a clot over time in the body. This process is a treatment option for clots that are not immediately life-threatening.

A nurse is preparing to administer a sublingual nitroglycerin tablet to a client who is reporting chest pain. For which of the following adverse effects should the nurse monitor after giving this medication? a. Hypotension b. Myalgia c. Diarrhea d. Ototoxicity

A. Hypotension---Nitroglycerin is a coronary vasodilator and antianginal agent. A major adverse effect of this medication is hypotension; therefore, blood pressure and pulse must be monitored before and after administration.

A nurse is monitoring a client for reperfusion following thrombolytic therapy to treat acute myocardial infarction (MI). Which of the following indicators should the nurse identify to confirm reperfusion? a. Ventricular dysrhythmias b. Appearance of Q waves c. Elevated ST segments d. Recurrence of chest pain

A. Ventricular dysrhythmias---The appearance of ventricular dysrhythmias following thrombolytic therapy is a sign of reperfusion of the coronary artery.

A nurse is caring for a client who had an onset of chest pain 24 hr ago. The nurse should identify that an increase in which of the following values is diagnostic of a myocardial infarction (MI)? A. Myoglobin B. C-reactive protein C. Creatine kinase-MB D. Homocysteine

C. Creatine kinase-MB Rationale: Creatine kinase-MB is the isoenzyme specific to the myocardium. Elevated creatine kinase-MB indicates myocardial muscle injury

A nurse is caring for a client who has a vitamin K deficiency. Which of the following manifestations should the nurse expect? a. Irregular bone formation b. Abnormal movements c. Blurred vision d. Excessive bruising

D. Excessive bruising--- The nurse should identify that excessive bruising can indicate bleeding under the skin. Vitamin K is needed by clotting factors to coagulate the blood. Therefore, a client who has a deficiency in vitamin K is at risk for excessive bruising and bleeding.

nurse is completing the admission assessment of a client who will undergo peripheral bypass graft surgery on the left leg. Which of the following is an expected finding? A. Rubor of the affected leg when elevated B. 3+ dorsal pedal pulse in left foot C. Thin, peeling toenails of left foot D. Report of intermittent claudication in the affected leg

D. Report of intermittent claudication in the affected leg

A nurse is applying antiembolic stockings for a client who has a history of deep vein thrombosis. Which of the following actions should the nurse take when applying the stockings? a. Roll the stocking partially down if too long b. Remove the stocking once per day c. Bunch and pull the stocking halfway up the calf d. turn the stocking inside out up to the heel before applying

D. Turn the stocking inside out up to the heel before applying--- The nurse should turn the stocking inside out up to the client's heel to make the application of the stocking easier and cause fewer constrictive wrinkles.

A nurse on a medical-surgical unit is caring for a client who reports pain in the jaw, back and shoulder, shortness of breath and nausea. Which of the following actions should the nurse perform first? 1. administer oxygen 2. begin ECG monitoring 3. ensure patent airway 4. assess pain

Ensure patent airway

A nurse is teaching a client who takes warfarin daily. Which of the following statements by the client indicates a need for further teaching? A) "I have started taking ginger root to treat my joint stiffness." B) "I take this medication at the same time each day." C) "I eat a green salad every night with dinner." D) "I had my INR checked three weeks ago."

I have started taking ginger root to treat my joint stiffness.

A nurse is caring for a client who is prescribed warfarin therapy for an artificial heart valve. Which of the following laboratory values should the nurse monitor for a therapeutic effect of warfarin? A) Hemoglobin (Hgb) B) Prothrombin (PT) C) Bleeding time D) Activated partial thromboplastin time (aPTT)

Prothrombin time (PT)

The nurse prepares a discharge teaching plan for a 44 yo male patient who has recently been diagnosed with coronary artery disease (CAD). Which risk factor should the nurse plan to focus on during the teaching session? a. Family cardiac history b. Hyperhomocysteinemia c. Elevated serum lipids d. Type A personality

c. Elevated serum lipids

When providing dietary instruction to a patient with hypertension, the nurse would advise the patient to restrict intake of which meat? a. Broiled fish b. Baked chicken breast c. Roasted duck d. Roasted turkey

c. Roasted duck Roasted duck is high in fat

A nurse is caring for a client who has atrial fibrillation and is receiving heparin. Which of the following findings is the nurse's priority? a. The client's EKG tracing shows irregular heart rate without P waves b. The client has an aPTT of 80 seconds c. The client experiences sudden weakness in one arm and leg d. The clients urine output is cloudy and odorless

c. The client experiences sudden weakness in one arm and leg a is incorrect, the EKG tracing isn't a priorityb is incorrect, the aPTT shows potential of being an issue, but isn't yetc is correct, a stroke could be happening in the brain and it's an actual issued is incorrect, irrelevant

Which one of these orders for a client who is in cardiogenic shock should the nurse question? a. give morphine IV b. monitor and document client's LOC c. monitor UO hourly d. start IV of NS at 150 ml/hour

d. start IV of NS at 150 ml/hour

The clinic nurse is evaluating a client who had coronary artery stenting through the right femoral artery a week previously and is taking metoprolol, clopidogrel, and aspirin. Which information reported by the client is most important to report to the health care provider? 1.Stools have been black in color. 2.Bruising is present at the right groin. 3.Home blood pressure today was 104/52 mm Hg. 4.Home radial pulse rate has been 55 to 60 beats/min.

1.Stools have been black in color.

While performing an admission assessment for a client, the nurse notes that the client has varicose veins with ulcerations and lower extremity edema with a report of a feeling of heaviness. Which of the following nursing diagnoses should the nurse identify as being the priority in the client's care? A) Impaired tissue perfusion B) Alteration in body image C) Alteration in activity tolerance D) Impaired Skin integrity

A) Impaired tissue perfusion

A nurse is monitoring a client who is on telemetry. Which of the following findings on the ECG strip should the nurse recognize as normal sinus rhythm? A) The P wave falls before the QRS complex. B) The T wave is the inverted position. C) The P-R interval measures 0.22 seconds. D) The QRS duration is 0.20 seconds.

A) The P wave falls before the QRS complex.

While reading a client's ECG tracing, the nurse should understand that the P wave reflects which of the following cardiac electrical activities? 1. ventricular depolarization 2. slow repolarization of ventricular Purkinje fibers 3. atrial depolarization 4. early ventricular repolarization

Atrial depolarization; typically initiated in the SA node.

A nurse is teaching a client who has hypertension and a new prescription for atenolol. Which of the following findings should the nurse include as adverse effects of this medication? A.) Bradycardia B.) Tremor C.) Cough D.) Constipation

A.) Atenolol is a beta-blocker, which slows the heart rate. The nurse should instruct the client to monitor his pulse rate and report bradycardia.

After teaching about ways to decrease risk factors for CAD, the nurse recognizes that further instruction is needed when the patient says a. "I can keep my blood pressure normal with medication." b. "I would like to add weight lifting to my exercise program." c. "I can change my diet to decrease my intake of saturated fats." d. "I will change my lifestyle to reduce activities that increase my stress."

B

A nurse is caring for a 72-year-old client who is to undergo a percutaneous balloon valvuloplasty. The client's daughter asks the nurse to explain the expected outcome of this procedure. Which of the following is an appropriate response by the nurse? A. "This will improve blood flow in your mother's coronary arteries." B. "This will permit your mother to resume her activities of daily living." C. "This will prolong your mother's life." D. "This will reverse the effects to the damaged area."

B. "This will permit your mother to resume her activities of daily living."

A nurse in the emergency department is caring for a client who took 3 nitroglycerin tablets sublingually for chest pain. The client reports relief from the chest pain but now he is experiencing a headache. Which of the following statements should the nurse make? a. "A headache is an indication of an allergy to the medication" b. "A headache is an expected adverse effect of the medication" c. "A headache indicates tolerance to the medication" d. "A headache is likely due to the anxiety about the chest pain"

B. "A headache is an expected adverse effect of the medication"--- The vasodilation nitroglycerin induces increases blood flow to the head & typically results in a headache.

A nurse is completing discharge teaching to a client who has heart failure and is encouraged to increase potassium in his diet. Which of the following statements by the client indicates understanding of the teaching? A. "I will consume more white rice." B. "I will eat more baked potatoes." C. "I will drink more grape juice." D. "I will use more powdered cocoa mixes."

B. "I will eat more baked potatoes."

A nurse is reinforcing discharge teaching with a client who has heart failure and is encouraged to increase potassium in his diet. Which of the following food selections should the nurse include has having the highest source of potassium? A. 1 medium apple B. 1 medium baked potato C. 1 slice toast with 1 tbsp peanut butter D. 1 large scrambled egg

B. A medium baked potato is best food source for K+ b/c it contains 926 mg K+ per serving

The nurse is monitoring a patient with suspected digoxin toxicity. Which assessment findings would be consistent with digoxin toxicity? Select all that apply. A. Diarrhea B. Anorexia C. Vomiting D. Dry cough E. Visual disturbances

B. Anorexia C. Vomiting E. Visual disturbances

A nurse is caring for a client following peripheral bypass graft surgery of the left lower extremity. Which of the following client findings pose an immediate concern? (Select all that apply.) A. Trace of bloody drainage on dressing B. Capillary refill of affected limb of 6 seconds C. Mottled appearance of the limb D. Throbbing pain of affected limb that is decreased following IV bolus analgesic E. Pulse of 2+ in the affected limb

B. Capillary refill of affected limb of 6 seconds C. Mottled appearance of the limb

A nurse in an emergency dept. is planning care for a client who is having an acute MI. The nurse should plan to administer which of the following medications after the initial acute phase to manage the client's pain and anxiety? A. Oxygen B. Morphine C. Aspirin D. NTG

B. Morphine

A client with valvular heart disease is at risk for developing left-sided heart failure. The nurse knows to monitor which of the following parameters to determine if the client has developed this disorder? 1. appetite 2. body weight 3. breath sounds 4. blood pressure

Breath sounds; manifestations of left-sided heart failure are crackles or wheezes

A nurse is caring for a client who has valvular heart disease and is at risk for developing left-sided heart failure. Which of the following manifestations should alert the nurse the client is developing this condition? Anorexia Weight gain Breathlessness Distended abdomen

Breathlessness Manifestations of left-sided heart failure include crackles or wheezes and breathlessness due to pulmonary congestion

A patient is recovering from an uncomplicated MI. Which rehabilitation guideline is a priority to include in the teaching plan? a. Refrain from sexual activity for a minimum of 3 weeks. b. Plan a diet program that aims for a 1- to 2-lb weight loss per week. c. Begin an exercise program that aims for at least 5 30-minute sessions per week. d. Consider the use of erectile agents and prophylactic NTG before engaging in sexual activity.

C

A nurse is caring for a client who has atrial fibrillation and receives digoxin daily. Before administering this medication, which of the following actions should the nurse take? A.) Offer the client a light snack. B.) Measure the client's blood pressure. C.) Measure the client's apical pulse. D.) Weigh the client.

C.) Digoxin decreases the heart rate, so the nurse should count the apical pulse for at least 1 min before administering. The nurse should hold the medication and notify the provider if the client's heart rate is below 60/min or if a change in heart rhythm is detected.

A nurse is teaching a client about following a low-cholesterol diet after coronary artery bypass grafting. Which of the following client food choices reflects the client's understanding of these dietary instructions? A.) Liver B.) Milk C.) Beans D.) Eggs

C.) Beans

A nurse is reviewing blood pressure classifications with a group of nurses at an inservice meeting. Which of the following should the nurse include as a risk factor for the development of hypertension? A.) High-density lipoprotein (HDL) level of 70 mg/dL B.) A diet high in potassium C.) Obstructive sleep apnea (OSA) D.) Taking benazepril

C.) The obstructed airway results in surges in the both the systolic and diastolic pressure during sleep and, in some clients, through the waking hours even when breathing is normal.

A nurse is educating a group of clients about the contraindications of warfarin therapy. Which of the following statements should the nurse include in the teaching? A.) "Clients who have glaucoma should not take warfarin." B.) "Clients who have rheumatoid arthritis should not take warfarin." C.) "Clients who are pregnant should not take warfarin." D.) "Clients who have hyperthyroidism should not take warfarin."

C.) Warfarin therapy is contraindicated in the pregnant client because it crosses the placenta and places the fetus at risk for bleeding.

A nurse in a clinic is caring for a client who has recently begun taking Warfarin. The nurse is reviewing potential drug and food interaction risk and should instruct the client to avoid which of the following? Cabbage Cantaloupe Green beans White beans

Cabbage Cabbage should be limited in the diet when taking warfarin, because it is rich in vitamin K.

While auscultating a client's heart sounds, the nurse hears turbulence between the S1 and S2 heart sounds. The nurse should document this finding as which of the following? 1. a cardiac murmur 2. third heart sound (S3) 3. an expected heart sound 4. a fourth heart sound (S4)

Cardiac murmur; they create a whooshing or swishing sound.

A nurse is providing teaching to a client who has a new prescription for hydrochlorothiazide 50 mg PO daily to treat hypertension. Which of the following instructions should the nurse include in the teaching? a. "Take hydrochlorothiazide as needed for edema." b. "Check your weight once each week." c. "Take hydrochlorothiazide on an empty stomach." c. "Take hydrochlorothiazide in the morning."

D. "Take hydrochlorothiazide in the morning."---The client should take hydrochlorothiazide in the morning to allow for diuresis during the day and to prevent nocturia.

A nurse is caring for a client who is receiving heparin by continuous IV infusion. Which of the following medications should the nurse plan to administer in the event of an overdose.

Protamine

A nurse is providing instruction to a new nurse about caring for clients who are receiving diuretic therapy to treat heart failure. The nurse should explain that which of the following medications puts clients at risk for both hyperkalemia and hyponatremia? a. furosemide b. hydrochlorothiazide c. metolazone d. spironolactone

Spironolactone

A nurse is providing teaching to a client who has HTN and a new prescription for hydrochlorothiazide. Which of the following instructions should the nurse provide? Weigh weekly to monitor therapeutic effect. Take the medication on an empty stomach. Take the medication early in the day. Muscle pain is an expected adverse effect.

Take the medication early in the day.The nurse should instruct the client to take hydrochlorothiazide early in the day to avoid nocturia.

A nurse is assessing a client who has a vitamin K deficiency. Which of the following indicates the nurse understands the effects of Vitamin K? 1. Vitamin K reverses warfarin toxicity 2. Vitamin K promotes fibrinogen formation 3. Vitamin K is produced in the gastric juices 4. Vitamin K normalizes the clotting factors reflected in aPTT

Vitamin K reverses warfarin toxicity; it promotes prothrombin formation in the liver. It normalizes the clotting factors reflected in PT, not in aPTT.

which of these clinical assessment findings would alert the nurse to intervene due to complications from bleeding following surgery. select all that apply a. CVP - 2 b. UO - 60 to 38 c. BP - 140/95 d. PAWP - 15 e. HR - 80 to 120

a. CVP - 2 b. UO - 60 to 38 e. HR - 80 to 120

A nurse is admitting a client who has a serum calcium level of 12.3mg and initiates cardiac monitoring. Which of the following findings should the nurse expect during the initial assessment? a. hyperactive deep tendon reflexes b. hyperactive bowel sounds c. lethargy d. Prolonged ST segment

c. lethargy A serium calcium of 12.3 mg/dL is above the expected range. The nurse should monitor the client for lethargy, generalized weakeness and confusion.

A patient presents to the emergency department with reports of chest pain for 3 hours. What component of his blood work is MOST clearly indicative of a myocardial infarction (MI)? a. Myoglobin b. C-reactive protein c. CK-MB d. Troponin

d. Troponin Troponin is the biomarker of choice in the diagnosis of MI, with sensitivity and specificity that exceed those of CK-MB and myoglobin. CRP levels are not used to diagnose acute MI

A nurse is preparing to teach a group of women in a community volunteer group about heart disease. What should the nurse include in the teaching plan? a. Women experience more symptoms of heart disease at a younger age than men b. Women are often less ill when presenting for treatment of heart disease c. Women are less likely to delay seeking treatment than men d. Women are more likely to have noncardiac symptoms of heart disease

d. Women are more likely to have non-cardiac symptoms of heart disease

a nurse is providing teaching to a client who has hypertension and a new prescription for captopril. Which of the following instructions should the nurse provide? a. don't use salt substitutes while taking this med b. take the med with food c. count your pulse rate before taking the med d. expect to gain weight while taking this med

do not use salt substitutes while taking this medication

A nurse is providing discharge teaching for a client who has pulmonary edema and is about to start taking furosemide. Which of the following instruction should the nurse make? a. take aspirin if headaches develop b.Eat foods that contain plenty potassium c. expect some swelling in the hands and feet d. take the med at bedtime

eat foods that contain plenty of potassium

A nurses are leaving a client who is immediately post-operative following hip arthroplasty. which of the following medication should the nurse plan to administer for DVT prophylaxis. Aspirin PO Enoxaparin subcutaneous Heparin infusion Warfarin PO

enoxaparin subcutaneous

A nurse is providing discharge teaching to a client who has a new prescription for Verapamil for Angina. Which of the following instruction should the nurse include? A. "Limit your fluid intake to mealtimes." B. "Do not take this medication on an empty stomach." C. "Increase your daily intake of dietary fiber." D. "You can expect swelling of the ankles while taking this medication."

increase your daily intake of dietary fiber

A nurse is teaching a middle-age client about hypertension. Which of the following information should the nurse include in the teaching? "Reaching your goal blood pressure will occur within 2 months." "Diuretics are the first type of medication to control hypertension." "Limit your alcohol consumption to three drinks a day." "Plan to lower saturated fats to 10 percent of your daily calorie intake."

"Diuretics are the first type of medication to control hypertension." The nurse should include in the teaching that diuretic medication is the first type of medication to control hypertension, by decreasing blood volume and lowering blood pressure.

A nurse is reviewing the lab results for a client who has a history of atherosclerosis and notes elevated cholesterol levels. Which of the following statements by the client indicates the nurse should plan follow-up teaching on a low-cholesterol diet? "I flavor my meat with lemon juice." "I eat two eggs for breakfast each morning." "I cook my food with canola oil." "I take an omega-3 supplement daily."

"I eat two eggs for breakfast each morning." Clients should limit egg yolks to two to three per week.

A nurse is providing teaching about a heart healthy diet to a group of clients with hypertension. Which of the following statements by one of the clients indicates a need for further teaching? "I may eat 10 ounces of lean protein each day." "Fresh fruits make a good snack option." "I will replace table salt with dried herbs." "I may thicken gravies with cornstarch as I cook."

"I may eat 10 ounces of lean protein each day." Lean meats should be limited to 5 to 6 oz per day. This statement by a client requires additional teaching.

A nurse is teaching a client who is taking atorvastatin daily. Which of the following statements by the client indicates an understanding of the teaching? A.) "I will avoid drinking grapefruit juice." B.) "I should take this medication without food." C.) "I should expect my stools to turn clay-colored." D.) "It is not necessary to have routine lab tests done."

A.) Grapefruits and grapefruit juice can reduce metabolism of atorvastatin, which increases the risk for toxicity.The client can take atorvastatin with or without food.

A nurse is caring for a client who has heart failure and a prescription for digoxin. Which of the following statements by the client indicates an adverse effect of the medication? A) "I can walk a mile a day." B) "I've had a backache for several days." C) "I am urinating more frequently." D) "I feel nauseated and have no appetite."

I feel nauseated and have no appetite.

A nurse is teaching a client who has a new prescription for clopidogrel. Which of the following instructions should the nurse include?

Take this med 3 times daily.

a nurse is teaching a client who has a new prescription for colesevelam to lower his lower density lipoprotein level. which of the following instructions should the nurse include? A Reduce fluid intake B. Chew tablets before swallowing C. Take this medication 4 hours after other medications D. Take this medication on an empty stomach.

Take this medication 4 hrs after other medications

A nurse is providing teaching for a client who has hypertension and a prescription change from metoprolol to metoprolol/hydrochlorothiazide. Which of the following statements by the client indicates an understanding of the teaching. a. "Now I will not have to diet to lose weight." b. "With the new medication, I should experience fewer side effects." c. "I will not have to do anything different because it is the same medication." d. "The extra letters after the name of medication means it is a stronger dose."

With the new medication, I should experience fewer side effects.

A nurse is caring for a client who has a prescription for Digoxin 0.25mg PO daily. The amount available is 0.125mg tab. The current vitals: BP- 144/96 HR- 54 RR- 18 Temp- 98.6. Which action should the nurse take?

Withhold the digoxin does for decreased pulse rate. The nurse should withhold the prescribed dose of digoxin as the heart rate is less than 60/min, and notify the provider.

A nurse is caring for a client who develops a ventricular fibrillation rhythm. The client is unresponsive, pulseless, and apneic. Which of the following actions is the nurse's priority? a. Defibrillation b. Airway management c. Epinephrine administration d. Amiodarone administration

a. Defibrillation a is correct, the patient is basically dead b is incorrect, you need to revive them first, then manage their airway when they're actually breathing c is incorrect, it's used in ACLS but not when the patient is dead d is incorrect, amiodarone is used in v-tach

A nurse is caring for a patient immediately following a transesophageal echocardiogram (TEE). Which assessments are appropriate for this patient? Select all that apply. a. Position patient supine with HOB flat b. Monitor vital signs and oxygen saturation c. Assess lower extremities for circulatory compromise d. Assess for return of gag reflex e. Assess groin for hematoma or bleeding

a. Position patient supine with HOB flat c. Assess lower extremities for circulatory compromise d. Assess for return of gag reflex Sedation is usually a throat-numbing spray. Vital signs and o2 saturation are important because of sedation used.

The nurse is performing an assessment for a patient undergoing radiation treatment for breast cancer. What position should the nurse place the patient to BEST auscultate for signs of acute pericarditis? a. Sitting and leaning forward b. Left lateral side-lying position c. HOB at a 45 degree angle d. Supine without a pillow

a. Sitting and leaning forward To auscultate a friction rub, they should be sitting like this and the nurse will hear it at the end of expiration

A nurse in an urgent care center is assessing a client who reports a sudden onset of irregular palpitations, fatigue, and dizziness. The nurse finds a rapid and irregular heart rate with a significant pulse deficit. Which of the following dysrhythmias should the nurse expect to find on the EKG? a. First-degree AV block b. Atrial fibrillation c. Sinus bradycardia d. Sinus tachycardia

b. Atrial fibrillation a is incorrect, rhythm would not be irregular b is correct, pulse deficit is a key sign of a-fib c is incorrect, rhythm would not be rapid or irregular d is incorrect, rhythm would not be irregular

The nurse instructs a 68 yo woman with hypercholesterolemia about natural lipid-lowering therapies. The nurse determines further teaching is necessary if the patient makes which statement? a. I should check with my physician before I start taking any herbal products b. I will take garlic instead of my prescription medication to reduce my cholesterol c. Herbal products do not go through as extensive testing as prescription drugs do d. Omega-3 fatty acids are helpful in reducing triglyceride levels

b. I will take garlic instead of my prescription medication to reduce my cholesterol

While assessing the cardiovascular status of a patient, the nurse performs auscultation. Which intervention should the nurse implement in the assessment during auscultation? a. Use the bell of the stethoscope when auscultating S1 & S2 b Palpate the radial pulse while auscultating the apical pulse c. Position the patient supine d. Ask the patient to hold their breath

b. Palpate the radial pulse while auscultating the apical pulse Sitting or side-lying is most appropriate for auscultation, but in order to detect any pulse deficits, it's important to check pulses at the same time

A patient is admitted with severe dyspnea, a history of heart failure, and chronic obstructive lung disease. Which diagnostic study would the nurse expect to be elevated if the cause of dyspnea was cardiac related? a. Serum homocysteine b. b-type natriuretic peptide (BNP) c. Serum potassium d. High-density lipoprotein (HDL)

b. b-type natriuretic peptide (BNP) elevation of BNP indicates the presence of heart failure. Elevations help to distinguish cardiac versus respiratory causes of dyspnea

which of these statements made by the charge nurse who is conducting an orientation program on the patho of cardiogenic shock indicates an appropriate understanding of this shock? a. occurs due to a mechanical blockage of the heart b. occurs due to a pump or heart failure c. occurs due to a decrease in intravascular volume d. occurs due to a widespread vasodilation and increased permeability

b. occurs due to a pump or heart failure

A nurse is admitting a client who has acute heart failure following myocardial infarction (MI). The nurse recognizes that which of the following prescriptions by the provider requires clarification? a. Morphine sulfate 2 mg IV bolus every 2 hr PRN pain b. Laboratory testing of serum potassium upon admission c. 0.9% NS IV at 50 mL/hr continuous d. Bumetanide (Bumex) 1 mg IV bolus every 12 hr

c. 0.9% NS IV at 50 mL/hr continuous a, b, and d are all okay orders to give. Bumex is a potassium wasting diuretic, so it's important to check potassium. You should always question fluids for a patient with heart failure.

A nurse is caring for an adult male client who is undergoing screening tests for atherosclerosis. Which of the following laboratory findings should the nurse identify as an increased risk for this disorder? a. Cholesterol level 195 mg/dL b. Elevated HDL levels c. Elevated LDL levels d. Triglyceride level 135 mg

c. Elevated LDL levels---An elevated LDL level increases a client's risk of atherosclerosis. The client's desirable LDL level is <100 mg/dL.

A client is prescribed nicotinic acid for hyperlipidemia and the nurse provides instructions to the client about the medication. Which statement by the client indicates an understanding of the instructions? 1. "It is not necessary to avoid the use of alcohol." 2. "The medication should be taken with meals to decrease flushing." 3. "Clay-colored stools are a common side effect and should not be of concern." 4. "Ibuprofen IB taken 30 minutes before the nicotinic acid should decrease the flushing."

4. "Ibuprofen IB taken 30 minutes before the nicotinic acid should decrease the flushing."

A nurse is caring for a client who has heart failure and a potassium level of 2.4 mEq/L. The nurse should identify which of the following medications as the cause of the client's low potassium level? A.) Furosemide B.) Nitroglycerin C.) Metoprolol D.) Spironolactone

A.) Furosemide is a loop (high-ceiling) diuretic that inhibits the reabsorption of sodium and chloride and results in diuresis, which decreases potassium through excretion in the distal nephrons. Hypokalemia is an adverse effect of furosemide.

The nurse is caring for a patient who is 2 days post MI. The patient reports that she is experiencing chest pain when she takes a deep breath. Which action would be a priority? a. Notify the provider STAT and obtain a 12-lead ECG. b. Obtain vital signs and auscultate for a pericardial friction rub. c. Apply high-flow O2 by face mask and auscultate breath sounds. d. Medicate the patient with as-needed analgesic and reevaluate in 30 minutes.

B

A nurse at a provider's office receives a phone call from a client who reports nausea and unrelieved chest pain after taking a nitroglycerin tablet 5 min ago. Which of the following is an appropriate response by the nurse? a. Tell the client to take an antacid b. Instruct the client to call 911 c. Tell the client to take another nitroglycerin tablet in 15 min d. Advise the client to come to office

B) Instruct the client to call 911-- The nurse should instruct the client to call 911 for transportaition to the emergency department because the client is having unstable angina or an acute MI.

The nurse is reinforcing teaching regarding diet to a client after a myocardial infarction. The nurse evaluates the reinforcement as effective if the client selects which of the following options? 1. barbecued beef, beans, potato chips, tossed salad 2. baked turkey, mashed potatoes, squash and salad 3. bread, fried fish patty, potato salad, cole slaw 4. grilled pork chops, biscuits and brown gravy, sliced tomato

Baked turkey, mashed potatoes, squash and salad; low sodium, low fat diet is a usual cardiac diet

A nurse is preparing an in-service presentation about assessing clients who are having an acute myocardial infarction (MI). What is the most common assessment finding with acute MI? a. Dyspnea b. Pain in the shoulder and left arm c. Substernal chest pain d. Palpitations

C. Substernal chest pain---Evidence-based practice indicates that the most common manifestation of acute MI is substernal chest pain that does not subside with rest or nitroglycerin. Therefore, nurses should make pain management with morphine a priority to reduce myocardial oxygen demand and increase oxygenation.

A nurse is monitoring a client who had a myocardial infarction. For which of the following complications should the nurse monitor in the first 24 hr? a. Infective endocarditis b. Pericarditis c. Ventricular dysrhythmias d. Pulmonary emboli

C. Ventricular dysrhythmias---- After a myocardial infarction, the electrical conduction system of the heart can be irritable and prone to dysrhythmias. Ischemic tissue caused by the infarction can also interfere with the normal conduction patterns of the heart's electrical system.

A nurse is assessing a client who is 85 years old. Which of the following findings should the nurse identify as a manifestation of myocardial infarction? a. Sudden hemoptysis b. Acute diarrhea c. Frontal headache d. Acute confusion

D. Acute confusion--- Acute confusion is a manifestation of myocardial infarction in clients age 65 or older. Other manifestations can include nausea, vomiting, dyspnea, diaphoresis, anxiety, dizziness, palpitations, and fatigue.

A nurse is caring for a client who has heart failure and is receiving IV furosemide. The nurse should monitor the client for which of the following electrolyte imbalances? Hypernatremia Hyperuricemia Hypercalcemia Hyperchloremia

Hyperuricemia The nurse should monitor the client who is receiving IV furosemide for hyperuricemia. The nurse should instruct the client to notify the provider for any tenderness or swelling of the joints.

A nurse is planning care for a client who has a suspected myocardial infarction. Which of the following should the nurse administer first? 1. nitroglycerin 2. aspirina 3. oxygen 4. morphine sulfate

Oxygen; use the airway, breathing, circulation approach

A nurse in a coronary care unit is admitting a client who has had CPR following a cardiac arrest. The client is receiving a lidocaine IV at 2 mg/min when the client asks the nurse why he is receiving that medication the nurse should explain that this has which of the following actions? a. Prevents dysrhythmias b. Slows intestinal motility c. Dissolves blood clots d. Relieves pain

prevents dysrythmias

The patient informs the nurse that he does not understand how there can be a blockage in the left anterior descending artery (LAD), but there is damage to the right ventricle. What is the BEST response by the nurse? a. The one vessel curves around from the left side to the right ventricle b. The right ventricle is supplied during systole primarily by the right coronary artery c. The LAD supplies blood to the left side of the heart and part of the right ventricle d. It is actually on your right side of the heart, but we call it the left anterior descending vessel

c. The LAD supplies blood to the left side of the heart and part of the right ventricle the lower part of the right ventricle receives blood flow from the left anterior descending artery as well as the right coronary artery during diastole

The nurse informs the patient that she must wear intermittent sequential compression stockings after a surgical procedure. What is an appropriate rationale for nurse to give to a patient for the use of the device? a. The socks maintain the blood flow to the legs while the patient is on bed rest b. The socks keep the legs warm while the patient is not moving much c. The socks provide compression of the veins to keep the blood moving back to the heart d. The socks keep the blood pressure down while the patient is stressed after surgery

c. The socks provide compression of the veins to keep the blood moving back to the heart. SCD's prevent blood from pooling in the lower extremities that could cause deep vein thrombosis

Which instruction by the nurse is given to a patient who is about to undergo Holter monitoring is MOST appropriate? a. You should refrain from exercising while wearing this monitor b. You may remove the monitor only to shower or bathe c. You will need to keep a diary of all your activities and symptoms d. You should connect the monitor whenever you feel symptoms

c. You will need to keep a diary of all your activities and symptoms Holter monitor is worn for 24 continuous hours while patient participates in normal activities and keeps journal of activities and symptoms, but shouldn't bathe while wearing monitor

A nurse is teaching a client who has a new prescription for Simvastatin. Which of the following instruction should the nurse include. A. "You should expect brown-colored urine." B. "You should avoid grapefruit juice." C. "You should monitor for ringing in the ears." D. "You should take the medication in the morning."

you should avoid grapefruit juice

A nurse is providing discharge instructions for a client with angina who has a prescription for sublingual nitroglycerin tablets. The nurse should teach the client that the nitroglycerin sublingual tablets have lost their potency when what happens? 1. Sublingual tingling is experienced. 2. The tablets are more than three months old. 3. The pain is unrelieved, but facial flushing is increased. 4. Onset of relief is delayed, but the duration of relief is unchanged.

2. The tablets are more than three months old.

At 10:00 am, a hospitalized client receives a new order for transesophageal echocardiography as soon as possible. Which action will the nurse take first? 1.Put the client on "nothing by mouth" (NPO) status. 2.Teach the client about the procedure. 3.Insert an IV catheter in the client's forearm. 4.Attach the client to a cardiac monitor.

4.Attach the client to a cardiac monitor.

When the nurse is monitoring a 53-year-old client who is undergoing a treadmill stress test, which finding will require the most immediate action? 1.Blood pressure of 152/88 mm Hg 2.Heart rate of 134 beats/min 3.Oxygen saturation of 91% 4.Chest pain level of 3 (on a scale of 0 to 10)

4.Chest pain level of 3 (on a scale of 0 to 10)

A nurse is caring for a client who has congestive heart failure and is taking digoxin daily. The client refused breakfast and is complaining of nausea and weakness. Which of the following actions should the nurse take first? A) Check the client's vital signs. B) Request a dietitian consult. C) Suggest that the client rests before eating the meal. D) Request an order for an antiemetic.

A) Check the client's vital signs.

A nurse is teaching a client who has a new diagnosis of A Fib. The nurse should instruct the client to monitor which of the following complications? A. Pulmonary embolism B. Bradycardia C. Peripheral vascular disease D. Hypertension.

A. Pulmonary embolism

A nurse is providing teaching for a client who has a new diagnosis of Angina Pectoris. The nurse should give the client which of the following information about Anginal pain? A. exertion and anxiety can trigger the pain B. the pain often radiates to the jaw or the back C. the pain usually lasts more than 20 min D. the pain persists with rest and organic nitrates

A. exertion and anxiety can trigger the pain

A nurse is providing instructions to a client who has a new prescription for sublingual nitroglycerin (Nitrostat) to treat angina pectoris. Which of the following instructions should the nurse include? A.) "Place the tablet under your tongue, and then take a small sip of water." B.) "The medication can take up to 15 minutes to take effect." C.) "Avoid taking the medication prior to exercising." D.) "Stop taking the medication and notify your provider if you develop a headache."

A.) A client who takes a sublingual medication should place it under his tongue. A sip of water can help the medication dissolve. The nurse should tell the client that the medication takes effect rapidly in 1-3 min.. The nurse should tell the client that the medication can be used to terminate an ongoing anginal attack as well as to prevent anginal pain prior to exertion. Therefore the client might need the medication prior to exercising. Headache is a common adverse effect of this medication that often dissipates with prolonged use. The client should continue to take the medication and take aspirin or acetaminophen for headache.

A nurse is teaching a client who has a new prescription for captopril. Which of the following instructions should the nurse include in the teaching? A.) Monitor for a cough. B.) Hold medication for heart rate less than 60/min. C.) Take this medication with food. D.) Avoid grapefruit juice.

A.) Captopril is an ACE inhibitor used to treat hypertension. The client should monitor and report a cough and dyspnea.

A nurse is providing discharge teaching to a client who has a new diagnosis of heart failure. Which of the following instructions should the nurse include in the teaching? A.) Exercise at least three times per week. B.) Take diuretics early in the morning and before bedtime. C.) Notify the provider of a weight gain of 0.5 kg (1 lb) in a week. D.) Take naproxen for generalized discomfort.

A.) Clients who have heart failure who remain active appear to have improved outcomes. Regular exercise strengthens the heart and cardiovascular system, thereby improving circulation and lowering blood pressure. The nurse should instruct the client to avoid the use of NSAIDs as these contribute to sodium and fluid retention, worsening the client's condition.

A nurse is caring for a client who is postoperative and is at risk for developing venous thromboembolism (VTE). The nurse should instruct the client to avoid which of the following unsafe actions? A) Elevating her feet B) Massaging her legs C) Flexing her ankles D) Ambulating soon after surgery

B) Massaging her legs

A nurse is caring for a client who is 4 hr postoperative following coronary artery bypass grafting (CABG) surgery. He is able to inspire 200 mL with the incentive spirometer, then refuses to cough because he is tired and it hurts too much. Which of the following is an appropriate nursing intervention? A. Allow the client to rest, and return in 1 hr. B. Administer IV bolus analgesic, and return in 15 min. C. Document the 200 mL as an appropriate inspired volume. D. Tell the client that he must try to cough if he does not want to get pneumonia.

B. Administer IV bolus analgesic, and return in 15 min.

The nurse is reinforcing teaching about a high-fiber diet for a cardiac patient. The patient asks what the purpose of the diet is. Which of the following replies by the nurse would be appropriate? A. "To increase absorption of nutrients in your diet." B. "It will reduce your heart's workload." C. "To prevent edema from developing." D. "To reduce your appetite."

B. Fiber helps prevent constipation & straining during bowel movements, which reduces cardiac workload

A nurse is reinforcing teaching with a client regarding reduction of risk factors for coronary artery disease. Which of the following statements by the client indicates an understanding of the teaching? Select all that apply. A. I should limit my exercise B. I must stop smoking C. I must stop drinking alcohol D. I should limit by intake of fast foods E. I need to monitor my weight

B. I must stop smoking D. I should limit by intake of fast foods E. I need to monitor my weight

A nurse is planning care for a client following a cardiac catheterization accessed through his femoral artery. Which of the following actions should the nurse plan to take? A. Instruct the client to perform range-of-motion exercises to his lower extremities. B. Perform neurovascular checks with vital signs. C. Ambulate the client 1 hr following the procedure. D. Restrict the client's fluid intake.

B. Perform neurovascular checks with vital signs.Rationale: The nurse should assess color, temperature, and pulse in the affected extremity and monitorthe client for neurovascular changes that can indicate a stroke, such as slurred speech andvisual disturbances.

A nurse is teaching with a group of nurses about the administration of Nitroglycerin. Which of the following routes of administration provides the most rapid onset for the client? A. Transdermal patch B. Sublingual C. Suspended-release D. Topical ointment

B. Sublingual

A nurse is caring for a client who has chronic venous insufficiency. The provider prescribed thigh-high compression stockings. The nurse should instruct the client to A. massage both legs firmly with lotion prior to applying the stockings. B. apply the stockings in the morning upon awakening and before getting out of bed. C. roll the stockings down to the knees if they will not stay up on the thighs. D. remove the stockings while out of bed for 1 hr, four times a day to allow the legs to rest.

B. apply the stockings in the morning upon awakening and before getting out of bed.

A nurse is caring for a client who is postoperative following an open reduction and internal fixation of a fracture femur. Which of the following actions is the most important for the nurse to complete in the postoperative period? A) Medicate the client for pain. B) Instruct the client on the use of crutches. C) Perform neurovascular checks of the extremities. D) Direct the client to perform exercises of the ankle and toes.

C) Perform neurovascular checks of the extremities.

A client who has a history of myocardial infarction (MI) is prescribed aspirin 325 mg. The nurse recognizes that the aspirin is given to which of the following actions of the medication? A) analgesic B) anti-inflammatory C) antiplatelet aggregate D) antipyretic

C) antiplatelet aggregate

A nurse is providing discharge instructions to a client following a cardiac cathertization. Which of the following information should the nurse include? A. "You can resume regular exercise as soon as tomorrow." B. "The dressing should be changed within 12 hours of the procedure." C. "You will notice a small hematoma at the incision site." D. "Pain medication will not be necessary."

C. "You will notice a small hematoma at the incision site." Rationale:Bruising and a small hematoma at the incision site are expected.

A nurse is caring for a client following an angioplasty that was inserted through the femoral artery. While turning the client, the nurse discovers blood underneath the client's lower back. The nurse should suspect A. retroperitoneal bleeding. B. cardiac tamponade. C. bleeding from the incisional site. D. heart failure.

C. bleeding from the incisional site.

A nurse is providing discharge instructions for a client who has congestive heart failure. Which of the following client statements indicates to the nurse that the teaching was effective? A.) "I will read food labels and limit my sodium to 4 grams per day." B.) "I should use naproxen to manage discomfort." C.) "I plan to slow down if I am tired the day after exercising." D.) "I will take my diuretic before sleep and drink fluids during the day."

C.) Clients who experience chest pain or dyspnea while exercising or experience fatigue the next day are probably advancing the activity too quickly and should slow down. The nurse should instruct the client to consume no more than 2 g of sodium per day. Excessive sodium intake increases fluid retention and the workload on the heart. A client who has heart failure should avoid the use of NSAIDs as these medications can cause sodium retention. The nurse should recommend the use of acetaminophen for the treatment of discomfort. Clients should be advised to take diuretics in the morning to avoid waking during the night for voiding.

A nurse is caring for a client who has congestive heart failure and is taking digoxin daily. The client refused breakfast and is complaining of nausea and weakness. Which of the following actions should the nurse take first? Check the client's vital signs. Request a dietitian consult. Suggest that the client rests before eating the meal. Request an order for an antiemetic.

Check the client's vital signs. It is possible that the client's nausea is secondary to digoxin toxicity. By obtaining vital signs, the nurse can assess for bradycardia, which is a symptom of digoxin toxicity. The nurse should withhold the medication and call the provider if the client's heart rate is less than 60 bpm.

A nurse is preparing to administer heparin subcutaneously to a client who has a deep vein thrombosis. Which of the following techniques should the nurse use? Cleanse the skin with an alcohol swab, insert the needle, aspirate, and inject the heparin. Cleanse the skin with an alcohol swab, insert the needle, aspirate, inject the heparin, and massage the site. Cleanse the skin with an alcohol swab, insert the needle, inject the heparin, and observe for bleeding. Cleanse the skin with an alcohol swab, insert the needle, inject the heparin, aspirate, and observe for bleeding.

Cleanse the skin with an alcohol swab, insert the needle, inject the heparin, and observe for bleeding.

A nurse is caring for a client who was recently admitted for atrial fibrillation. Which of the following assessment findings should the nurse report immediately? 1. ECG reflects A. fib with a heart rate of 112/min 2. aPTT result of 40 seconds 3. Client's skin is cool and clammy 4. Urine output is cloudy and odorous

Client's skin is cool and clammy; indicates poor vascular perfusion. Should immediately be assessed for additional signs of shock and notify provider immediately. Normal aPTT is 30-40 seconds.

A client, newly prescribed a low-sodium diet due to hypertension, is asking for help with meal choices. The client provides four meal choices, which are favorites. Which selection would be best? a) Toasted cheese sandwich on whole wheat toast with tomato soup b) Creamed chipped beef over with mashed potatoes c) Hot dog with ketchup on whole wheat bun d) Green pepper stuffed with fresh tomatoes and chicken

D. green pepper stuffed with fresh tomatoes and chicken

A nurse is reviewing lab results of a client who has A Fib and is taking Warfarin. For which of the following results should the nurse notify the provider? A. Hct 44% B. Hgb 16 C. Platelets 190,000 D. PT 55 seconds

D. the expected range for PT is 10-12 seconds

A nurse is providing teaching to a client who is taking warfarin about monitoring its therapeutic effects. Which of the following explanations should the nurse provide about the international normalized ratio (INR) test? A.) "The INR also monitors heparin therapy if the provider switches the medication prescription." B.) "The INR is the only test available for anticoagulant therapy monitoring." C.) "You will only need the test twice per month." D.) "The INR is a standardized test that eliminates the variations between laboratories reports in prothrombin times."

D.) The INR is a standardized test, which means that the result will be the same, no matter which laboratory performs it.

A nurse is completing a medical interview with a client who has elevated cholesterol levels and takes warfarin. The nurse should recognize that which of the following actions by the client can potentiate the effects of warfarin? A.) The client follows a low-fat diet to reduce cholesterol. B.) The client drinks a glass of grapefruit juice every day. C.) The client sprinkles flax seeds on food 1 hr before taking the anticoagulant. D.) The client uses garlic to lower cholesterol levels.

D.) The nurse should recognize that garlic can potentiate the action of the warfarin.

A nurse is preparing an in-service about the various supplements clients might use. Which of the following herbal supplements should the nurse include as potentially increasing the anticoagulant effects of aspirin and other oral anticoagulants? a. Valerian b. Feverfew c. Milk Thistle d. Saw Palmetto

Feverfew

A nurse at a provider's office receives a phone call from a client who reports nausea and unrelieved chest pain after taking a nitroglycerin tablet 5 minutes ago. Which of the following is an appropriate response by the nurse? Tell the client to take an aspirin. Instruct the client to call 911. Have the client take another nitroglycerin tablet in 15 min. Advise the client to come to office.

Instruct the client to call 911.

A nurse in the emergency dept. is caring for a client who reports chest pressure and SOB. Which of the following lab tests should the nurse anticipate the provider to order? a. Troponin I b. Lipase c. BNP d. AST

Troponin I

A nurse in the emergency department is caring for a client who took 3 nitroglycerin tablets sublingually for chest pain. The client reports relief form the chest pain but not he is experiencing a headache. Which of the following statements should the nurse make? A. "A headache is an indication of an allergy to the medication." B. "A headache is an expected adverse effect of the medication." C. "A headache indicates tolerance to the medication." D. "A headache is likely due to the anxiety about the chest pain."

"A headache is an expected adverse effect of the medication"

A nurse is assessing a client who has deep-vein thrombosis in her left calf. Which of the following manifestations should the nurse expect to find? (Select all that apply.) a. Hardening along the blood vessel b. Absence of a peripheral pulse c. Tenderness in the calf d. Cool skin on the leg e. Increased leg circumference

A. Hardening along the blood vessel C. Tenderness in the calf E. Increased leg circumference--- Deep-vein thrombosis can cause hardening along the affected blood vessel and prominence of superficial veins, pain or tenderness in the calf, and an increase in the circumference of the leg due to swelling.

A nurse is assessing a client who has heart failure and is taking daily furosemide. The client's apical pulse is weak and irregular. The nurse should identify these findings as manifestations of which of the following electrolyte imbalances? a. Hypokalemia b. Hypophosphatemia c. Hypercalcemia d. Hypermagnesemia

A. Hypokalemia---- Furosemide can cause the loss of potassium, sodium, calcium, and magnesium. Manifestations of hypokalemia can include shallow respirations, muscle weakness, lethargy, and ectopic heartbeats.

A nurse is examining the ECG of a client who is having an acute myocardial infarction. The nurse should identify that the elevated ST segments on the ECG indicate which of the following alterations? a. Necrosis b. Hypokalemia c. Hypomagnesemia d. Insufficiency

A. Necrosis---- ST-segment elevation during an acute myocardial infarction indicates necrosis. This ECG change reflects a clot at the site of injury. Therefore, the client requires immediate revascularization of the artery.

A nurse is caring for a client with heart failure whose telemetry reading displays a flattening of the T wave. Which of the following laboratory results should the nurse anticipate as the cause of this ECG change? a. Potassium 2.8 mEq/L b. Digoxin level 0.7 ng/mL c. Hemoglobin 9.8 g/dL d. Calcium 8.0 mg

A. Potassium 2.8 mEq/L---A flattened T wave or the development of U waves is indicative of a low potassium level.

A nurse on a telemetry unit is caring for a client who had a myocardial infarction. The client states, "All this equipment is making me nervous." Which of the following responses should the nurse offer? a. "You won't need the equipment for very long." b. "All of this equipment can be frightening." c. "Why does the equipment bother you?" d. "Let me tell you about what each machine does."

B. "All of this equipment can be frightening."---This statement is therapeutic because the nurse is reflecting the client's statement. The client is feeling fearful, and this response shows the nurse understands those feelings, which will encourage the client to communicate more.

A nurse is providing teaching for a client who has hypertension and a prescription change from metoprolol/hydrochlorothiazide. Which of the following statements by the client indicates an understanding of the teaching? a. "Now I will not have to diet to lose weight." b. "With the new medication, I should experience fewer side effects." c. "I will not have to do anything different because it is the same medication." d. "The extra letters after the name of medication means it is a stronger dose."

B. "With the new medication, I should experience fewer side effects."--- The client has states an understanding of the purpose of the addition of the hydrochlorothiazide to the metoprolol dosage. When used in combo with thiazide diuretic, a lower dose of the beta-blocker can be used. The benefit is there are fewer side effects when beta-blockers (and other antihypertensives) are used in lower dosages.

A nurse is reviewing the medical record of a client who has hypertension and a new prescription for metoprolol. Which of the following findings should the nurse investigate further? a. Diet-controlled type 2 diabetes mellitus b. A hx of left-sided heart failure c. A concurrent prescription for tadalafil d. Recently treated bilateral pneumonia

B. A hx of left-sided heart failure--- The nurse should further investigate the client's hx of heart failure. Although metoprolol can be used to treat heart failure, it can also cause heart failure, so this medication should be used with great caution with a client who has a hx of heart failure. The nurse should teach the client to watch for signs of increasing left-sided heart failure, such as shortness of breath & wt gain indicating fluid retention, & report these findings to the provider.

A nurse is assessing a client who has peripheral vascular disease and a venous ulcer on the right ankle. Which of the following findings should the nurse expect in the client's affected extremity? a. Absent pedal pulses b. Ankle swelling c. Hair loss d. Skin atrophy

B. Ankle swelling--- The nurse should identify that swelling of the ankle is a manifestation of venous insufficiency due to poor venous return. Other manifestations can include brown pigmentations and cellulitis.

A nurse is providing discharge teaching to a client who has venous thrombosis and a prescription for warfarin. Which of the following instructions should the nurse include in the teaching? a. Take ibuprofen as needed for headaches or other minor pains b. Carry a medical alert ID card c. Report to the laboratory weekly to have blood drawn for aPTT d. Increase intake of dark green vegetables

B. Carry a medical alert ID card---A client who is taking warfarin is at increased risk for bleeding. In the case of an emergency, any medical personnel must be aware of the client's medication history.

A nurse is preparing an in-service presentation about the management of myocardial infarction (MI). Death following MI is often a result of which of the following complications? a. Cardiogenic shock b. Dysrhythmias c. Heart failure d. Pulmonary edema

B. Dysrhythmias---- According to evidence-based practice, dysrhythmias (specifically ventricular fibrillation) are the most common cause of death following MI. Therefore, nurses should monitor clients' ECGs carefully for dysrhythmias and report and treat them immediately.

A nurse is caring for a client who has severely elevated blood pressure. Which of the following findings should the nurse identify as a manifestation of hypertension? a. Vertigo b. Epistaxis c. Exophthalmos d. Spondylolisthesis

B. Epistaxis---Epistaxis (a nosebleed) is a manifestation of elevated blood pressure. Hypertension is often asymptomatic, but when it is severely elevated, it can also cause headaches, dizziness, facial flushing, and fainting.

A nurse completing an assessment on a client. Which of the following findings should the nurse identify as a risk factor for coronary artery disease? (Select all that apply.) a. Hypothyroidism b. Hypertension c. Diabetes mellitus d. Hyperlipidemia e. Tobacco smoking

B. Hypertension C. Diabetes mellitus D. Hyperlipidemia E. Tobacco smoking--- A client who has hypertension, diabetes mellitus, hyperlipidemia, or a history of smoking tobacco is at risk for coronary artery disease (CAD). Hypertension and hyperlipidemia can be controlled by diet and exercise, along with medication if needed. Diabetes can cause damage to large and small blood vessels, which leads to poor perfusion, cell death, and organ damage. Diabetes mellitus can be managed by monitoring glucose levels and implementing diet and exercise recommendations. Cholesterol levels, such as total HDL and LDL levels, should be monitored since elevated total serum cholesterol levels increase the risk of a myocardial infarction. Finally, smoking accelerates the rate of the narrowing of the coronary arteries and increases the risk of clot formation. Smoking cessation classes or other forms of treatment can be offered to help the client quit smoking.

A nurse is caring for a client who has unstable angina. The nurse should anticipate a prescription from the provider for which of the following medications? a. Epinephrine b. Nitroglycerin c. Lidocaine d. Atropine

B. Nitroglycerin--- The nurse should anticipate a prescription for nitroglycerin, which is indicated for a client who has unstable angina. Nitroglycerin is an organic nitrate and a vasodilator that acts by relaxing or preventing spasms in the coronary arteries, thereby decreasing the oxygen demand of the heart along with ventricular filling.

A nurse is caring for a client who is having a possible myocardial infarction (MI). Which of the following findings should the nurse identify as an associated manifestation of an MI? a. Headache b. Hemoptysis c. Nausea d. Diarrhea

C. Nausea---Nausea is an associated manifestation of MI. Manifestations of MI include chest pain and pain in the jaw, shoulder, or abdomen.

A nurse is admitting a client who is scheduled to undergo a cardiac catheterization. The client says, "My coworker died last week from a heart attack." Which of the following responses should the nurse offer? a. "Your provider will not let that happen because she knows how to treat your condition." b. "Do you think the same thing might happen to you?" c. "You appear to be feeling anxious." d. "Has anyone in your family had a heart attack?"

C. "You appear to be feeling anxious."---The nurse is sharing observations that will encourage the client to be more specific about these feelings.

A nurse is assessing a 6-month-old infant following a cardiac catheterization. Which of the following findings should the nurse report to the provider? a. Temperature 37.5°C (99.5°F) b. Apical pulse rate 140/min c. BP 86/40 mmHg d. Respiratory rate 32/min

C. BP 86/40 mmHg---A BP of 86/40 mmHg is indicative of hypotension and bleeding in a 6-month-old infant and should be immediately reported to the provider.

A nurse is reviewing the medical record of a client who is receiving hydrochlorothiazide (HCTZ). The nurse should expect to find an improvement in which of the following conditions as a result of this medication? a. Gouty arthritis b. Dehydration c. Diabetes insipidus d. Hypokalemia

C. Diabetes insipidus--- A thiazide diuretic such as HCTZ is administered to treat diabetes insipidus. Diabetes insipidus is a condition in which there is an overproduction of urine. Thiazides reduce urine production by 30% to 50%.

A nurse in a clinic is assessing the lower extremities and ankles of a client who has a history of peripheral arterial disease. Which of the following findings should the nurse expect? a. Pitting edema b. Areas of reddish-brown pigmentation c. Dry, pale skin with minimal body hair d. Sunburned appearance with desquamation

C. Dry, pale skin with minimal body hair--- A client who has peripheral arterial disease can display dry, scaly, pale, or mottled skin with minimal body hair because of narrowing of the arteries in the legs and feet. This causes a decrease in blood flow to the distal extremities, which can lead to tissue damage. Common manifestations are intermittent claudication (leg pain with exercise), cold or numb feet at rest, loss of hair on the lower legs, and weakened pulses.

A nurse is preparing to administer digoxin to a client. Which of the following findings should the nurse identify as a contraindication to the client receiving this medication? a. Blood pressure 180/70 mmHg b. Oxygen saturation rate 94% c. Heart rate 51/min d. Respiratory rate 21/min

C. Heart rate 51/min---- The nurse should identify that if the client's heart rate is less than 60/min, the medication should be withheld, and the provider should be notified.

A nurse in the emergency room is caring for a client who presents with manifestations that indicate a myocardial infarction. Which of the following prescriptions should the nurse take first? a. Attach the leads for a 12-lead ECG b. Obtain a blood sample c. Initiate O2 therapy d. Insert the IV catheter

C. Initiate O2 therapy--- The greatest risk to the client's safety is myocardial ischemia and cellular death; therefore, the priority action the nurse should take is to admin. O2 to help minimize this possibility.

A nurse is caring for a client who reports crushing chest pain. The nurse reviews the client's ECG results and notes ST changes. Which of the following medications should the nurse administer? a. Simvastatin b. Furosemide c. Nitroglycerin d. Sildenafil

C. Nitroglycerin---The nurse should identify the need to administer nitroglycerin, which is used to treat angina. Nitroglycerin acts directly on vascular smooth muscle to promote vasodilation.

A nurse is caring for a client who reports a new onset of severe chest pain. Which of the following actions should the nurse take to determine if the client is experiencing a MI? a. Check the client's BP b. Auscultate heart tones c. Perform a 12-lead ECG d. Determine if pain radiates to the left arm

C. Performs a 12-lead ECG--- The nurse should perform a 12-lead ECG when a client complains of chest pain to determine if the client is experiencing a MI.

A nurse is caring for a client who came to the emergency department reporting chest pain. The provider suspects a MI. While waiting for the troponin levels report, the client asks what this blood test will show. Which of the following explanations should the nurse provide the client? a. Troponin is an enzyme that indicates damage to the brain, heart, & skeletal muscle tissue b. Troponin is a lipid whose levels reflect the risk for coronary artery disease c. Troponin is a heart muscle protein that appears in the bloodstream when there is damage to the heart d. Troponin is a protein that helps transport O2 throughout the body

C. Troponin is a heart muscle protein that appears in the bloodstream when there is damage to the heart--- Troponin is a myocardial muscle protein that releases into the bloodstream when there is injury to the myocardial muscle. Troponin levels are specific point-of-care testing for clients who are having a myocardial infarction.

A nurse is assessing a client who has heart failure and is receiving digoxin. Which of the following findings should indicate to the nurse the client is experiencing digoxin toxicity? a. Suppression of dysrhythmias b. Increased atrioventricular (AV) conduction c. Visual disturbances d. Weight gain

C. Visual disturbances----The nurse should recognize that nausea, vomiting, abdominal discomfort, fatigue, and visual disturbances are common manifestations that can indicate that the client is experiencing digoxin toxicity.

A nurse is providing teaching for a client who has a new prescription for nitroglycerin administered through a transdermal patch. Which of the following client statements indicates an understanding of the teaching? a. "I need to wear the patch continuously for it to be effective." b. "I will stop using the patch immediately if it gives me a headache." c. "I should change the patch whenever I have chest pain." d. "I need to rotate the location of my patch every few days."

D. "I need to rotate the location of my patch every few days."---The nitroglycerin patch should be rotated to different hairless areas of the client's body every few days to avoid local skin irritation.

A nurse is providing discharge teaching to a client who has heart failure and a prescription for digoxin 0.125 mg PO daily and furosemide 20 mg PO daily. Which of the following statements by the client indicates an understanding of the teaching? a. "I know that blurred vision is expected to happen while I'm taking digoxin." b. "I will measure my urine output each day and document it in my diary." c. "I will skip a dose of my digoxin if my resting heart rate is below 72 beats per minute." d. "I will eat fruits and vegetables that have a high potassium content every day."

D. "I will eat fruits and vegetables that have a high potassium content every day."--- Hypokalemia is an adverse effect of diuretic therapy. Because the client is taking digoxin, it is important to maintain a potassium level between 3.5 to 5.0 mg/dL to avoid digoxin toxicity.

A nurse is teaching a 70-year-old client about risk factors for heart failure. The client has mild asthma, diabetes mellitus, and coronary artery disease. Which of the following statements by the client indicates an understanding of the teaching? a. "My diabetes will not increase my risk of heart failure." b. "My asthma makes it more likely for me to have heart failure." c. "My age does not increase my risk of heart failure." d. "My coronary artery disease is a risk factor for heart failure."

D. "My coronary artery disease is a risk factor for heart failure."---- Coronary artery disease is a primary risk factor for the development of heart failure. Other risk factors include hypertension, cardiomyopathy, tobacco use, family history, and hyperthyroidism.

A nurse is preparing to administer verapamil by IV bolus to a client who is having cardiac dysrhythmias. for which of the following adverse effects should the nurse monitor when giving this medication? a. hyperthermia b. hypotension c. ototoxicity d. muscle pain

Hypotension

A nurse is providing teaching to a client who is scheduled to start taking hydrochlorothiazide for hypertension. The nurse instructs the client to eat foods that are rich in potassium. Which of the following statements by the client indicates an understanding of the teaching? a. "This medication will not work unless I have enough potassium." b. "Potassium will increase the therapeutic effect of my blood pressure medication." c. "Potassium will lower my blood pressure. d. "This medication can cause a loss of potassium."

D. "This medication can cause a loss of potassium."--- Hydrochlorothiazide can result in hypokalemia caused by excessive potassium excretion from the kidneys. The client should supplement his diet with potassium-rich foods to avoid the occurrence of hypokalemia. Foods that are high in potassium include bananas, raisins, baked potatoes, pumpkins, and milk.

A nurse is caring for a child who has epistaxis. Which of the following actions should the nurse take? a. Administer aspirin b. Tilt the child's head back and apply pressure c. Have the child lie down and rest d. Apply continuous pressure to the lower part of the child's nose

D. Apply continuous pressure to the lower part of the child's nose----With the child sitting up and breathing through the mouth, the nurse should apply continuous pressure with the thumb and forefinger to the soft lower area of the nose for 10 minutes. Most bleeding from the nose stops within this period.

A nurse is caring for a client who has thrombocytopenia and develops epistaxis. Which of the following actions should the nurse take? a. Have the client gently blow clots from the nose every 5 min b. Instruct the client to sit with his head hyperextended c. Apply ice compresses to the back of the client's neck d. Apply lateral pressure to the client's nose for 10 min

D. Apply lateral pressure to the client's nose for 10 min----The nurse should apply direct, lateral pressure to the nose for 10 minutes to control epistaxis. If after 10 minutes the epistaxis continues, the client might require nasal packing or other interventions.

A nurse is caring for a client who has congestive heart failure and is taking digoxin. The client reports nausea and refuses to eat breakfast. Which of the following actions should the nurse take first? a. Encourage the client to eat the toast on the breakfast tray b. Administer an antiemetic c. Inform the client's provider d. Check the client's apical pulse

D. Check the client's apical pulse---- Nausea, anorexia, fatigue, visual effects, and cardiac dysrhythmias (often caused by a slow pulse rate) are possible findings in digoxin toxicity. Caring for this client requires the application of the nursing process priority-setting framework. The nurse can use the nursing process to plan client care and prioritize nursing actions. Each step of the nursing process builds on the previous step, beginning with an assessment. Before the nurse can formulate a plan of action, implement a nursing intervention, or notify the provider about a change in the client's status, the nurse must first collect adequate data from the client. Assessing will provide the nurse with the knowledge to make an appropriate decision.

A nurse is completing an assessment for a client who has a history of unstable angina. Which of the following findings should the nurse expect? a. Chest pain is relieved soon after resting. b. Nitroglycerin relieves chest pain. c. Physical exertion does not precipitate chest pain. d. Chest pain lasts for longer than 15 min.

D. Chest pain lasts for longer than 15 min--- A client who has unstable angina will have chest pain lasting longer than 15 minutes. This is due to reduced blood flow in a coronary artery from atherosclerotic plaque and thrombus formation causing partial arterial obstruction or from an artery spasm.

A nurse is caring for a client who is taking lisinopril. Which of the following outcomes indicates a therapeutic effect of the medication? A) Decreased blood pressure B) Increase of HDL cholesterol C) Prevention of bipolar manic episodes D) Improved sexual function

Decreased blood pressure

A client is diagnosed with an ST segment elevation myocardial infarction (STEMI) and is receiving a tissue plasminogen activator, alteplase. Which action is a priority nursing intervention? 1. Monitor for kidney failure. 2. Monitor psychosocial status. 3. Monitor for signs of bleeding. 4. Have heparin sodium available.

3. Monitor for signs of bleeding.

A nurse is caring for a client who has hypertension and develops epistaxis. Which of the following actions should the nurse take? (SATA). a. Apple pressure to the nares b. Place ice to the bridge of the client's nose c. Instruct the client to blow his nose d. Tilt the client's head backward e. Move the client into high-fowler's position

A B E

Which nursing responsibilities are priorities when caring for a patient returning from a cardiac catheterization (select all that apply)? a. Monitoring vital signs and ECG b. Checking the catheter insertion site and distal pulses c. Helping the patient to ambulate to the bathroom to void d. Telling the patient that he will be sleepy from the general anesthesia e. Teaching the patient about the risks of the radioactive isotope injection

A B

A patient has a severe blockage in his right coronary artery. Which heart structures are most likely to be affected by this blockage (select all that apply)? a. AV node b. Left ventricle c. Coronary sinus d. Right ventricle e. Pulmonic valve

A, B, D

A client receiving thrombolytic therapy with a continuous infusion of alteplase suddenly becomes extremely anxious and complains of itching. The nurse hears stridor and notes generalized urticaria and hypotension. Which nursing action is the priority? 1. Administer oxygen and protamine sulfate. 2. Cut the infusion rate in half and sit the client up in bed. 3. Stop the infusion and call for the Rapid Response Team (RRT). 4. Administer diphenhydramine and epinephrine and continue the infusion.

3. Stop the infusion and call for the Rapid Response Team (RRT).

The nurse is monitoring a client who is taking propranolol. Which assessment finding indicates a potential adverse complication associated with this medication? 1. The development of complaints of insomnia 2. The development of audible expiratory wheezes 3. A baseline blood pressure of 150/80 mm Hg followed by a blood pressure of 138/72 mm Hg after 2 doses of the medication 4. A baseline resting heart rate of 88 beats/minute followed by a resting heart rate of 72 beats/minute after 2 doses of the medication

2. The development of audible expiratory wheezes

A client being treated for heart failure is administered intravenous bumetanide. Which outcome indicates that the medication has achieved the expected effect? 1. Cough becomes productive of frothy pink sputum. 2. Urine output increases from 10 mL/hour to greater than 50 mL hourly. 3. The serum potassium level changes from 3.8 to 3.1 mEq/L (3.8 to 3.1 mmol/L). 4. B-type natriuretic peptide (BNP) factor increases from 200 to 262 pg/mL (200 to 262 ng/L).

2. Urine output increases from 10 mL/hour to greater than 50 mL hourly.

The nurse should report which assessment finding to the health care provider (HCP) before initiating thrombolytic therapy in a client with pulmonary embolism? 1. Adventitious breath sounds 2. Temperature of 99.4 °F (37.4 °C) orally 3. Blood pressure of 198/110 mm Hg 4. Respiratory rate of 28 breaths/minute

3. Blood pressure of 198/110 mm Hg

A client with atrial fibrillation is receiving a continuous heparin infusion at 1000 units/hour. The nurse determines that the client is receiving the therapeutic effect based on which results? 1. Prothrombin time of 12.5 seconds 2. Activated partial thromboplastin time of 60 seconds 3. Activated partial thromboplastin time of 28 seconds 4. Activated partial thromboplastin time longer than 120 seconds

2. Activated partial thromboplastin time of 60 seconds

A client with a clot in the right atrium is receiving a heparin sodium infusion at 1000 units/hour and warfarin sodium 7.5 mg at 5:00 p.m. daily. The morning laboratory results are as follows: activated partial thromboplastin time (aPTT), 32 seconds; international normalized ratio (INR), 1.3. The nurse should take which action based on the client's laboratory results? 1. Collaborate with the health care provider (HCP) to discontinue the heparin infusion and administer the warfarin sodium as prescribed. 2. Collaborate with the HCP to obtain a prescription to increase the heparin infusion and administer the warfarin sodium as prescribed. 3. Collaborate with the HCP to withhold the warfarin sodium since the client is receiving a heparin infusion and the aPTT is within the therapeutic range. 4. Collaborate with the HCP to continue the heparin infusion at the same rate and to discuss use of dabigatran etexilate in place of warfarin sodium.

2. Collaborate with the HCP to obtain a prescription to increase the heparin infusion and administer the warfarin sodium as prescribed

The nurse is monitoring a client who is taking digoxin for adverse effects. Which findings are characteristic of digoxin toxicity? Select all that apply. 1. Tremors 2. Diarrhea 3. Irritability 4. Blurred vision 5. Nausea and vomiting

2. Diarrhea 4. Blurred vision 5. Nausea and vomiting

Intravenous heparin therapy is prescribed for a client. While implementing this prescription, the nurse ensures that which medication is available on the nursing unit? 1. Vitamin K 2. Protamine sulfate 3. Potassium chloride 4. Aminocaproic acid

2. Protamine sulfate

The nurse is planning to administer hydrochlorothiazide to a client. The nurse should monitor for which adverse effects related to the administration of this medication? 1. Hypouricemia, hyperkalemia 2. Increased risk of osteoporosis 3. Hypokalemia, hyperglycemia, sulfa allergy 4. Hyperkalemia, hypoglycemia, penicillin allergy

3. Hypokalemia, hyperglycemia, sulfa allergy

The nurse provides discharge instructions to a client who is taking warfarin sodium. Which statement, by the client, reflects the need for further teaching? 1. "I will avoid alcohol consumption." 2. "I will take my pills every day at the same time." 3. "I have already called my family to pick up a MedicAlert bracelet." 4. "I will take coated aspirin for my headaches because it will coat my stomach."

4. "I will take coated aspirin for my headaches because it will coat my stomach."

The home health care nurse is visiting a client with elevated triglyceride levels and a serum cholesterol level of 398 mg/dL (10 mmol/L). The client is taking cholestyramine and the nurse teaches the client about the medication. Which statement, by the client, indicates the need for further teaching? 1. "Constipation and bloating might be a problem." 2. "I'll continue to watch my diet and reduce my fats." 3. "Walking a mile each day will help the whole process." 4. "I'll continue my nicotinic acid from the health food store."

4. "I'll continue my nicotinic acid from the health food store."

A client who is receiving digoxin daily has a serum potassium level of 3 mEq/L (3 mmol/L) and is complaining of anorexia. The health care provider prescribes a serum digoxin level to be done. The nurse checks the results and should expect to note which level that is outside of the therapeutic range? 1. 0.3 ng/mL 2. 0.5 ng/mL 3. 0.8 ng/mL 4. 1.0 ng/mL

4. 1.0 ng/mL

A client is being treated with procainamide for a cardiac dysrhythmia. Following intravenous administration of the medication, the client complains of dizziness. What intervention should the nurse take first? 1. Measure the heart rate on the rhythm strip. 2. Administer prescribed nitroglycerin tablets. 3. Obtain a 12-lead electrocardiogram immediately. 4. Auscultate the client's apical pulse and obtain a blood pressure.

4. Auscultate the client's apical pulse and obtain a blood pressure

Prior to administering a client's daily dose of digoxin, the nurse reviews the client's laboratory data and notes the following results: serum calcium, 9.8 mg/dL (2.45 mmol/L); serum magnesium, 1.0 mEq/L (0.5 mmol/L); serum potassium, 4.1 mEq/L (4.1 mmol/L); serum creatinine, 0.9 mg/dL (79.5 mcmol/L). Which result should alert the nurse that the client is at risk for digoxin toxicity? 1. Serum calcium level 2. Serum potassium level 3. Serum creatinine level 4. Serum magnesium level

4. Serum magnesium level

A nurse is preparing a client for cardiac catheterization. Which of the following pieces of information should the nurse give the client before the procedure? (Select all that apply.) a. "You'll have to lie flat for several hours after the procedure." b. "You'll receive medication to relax you before the procedure." c. "You'll feel a cool sensation after the injection of the dye." d. "You'll have to keep your leg straight after the procedure." e. "You'll have to limit the amount of fluid you drink for the first 24 hr."

A. "You'll have to lie flat for several hours after the procedure." B. "You'll receive medication to relax you before the procedure." D. "You'll have to keep your leg straight after the procedure."---Depending on the provider's prescription, the client should remain flat or with the head of the bed elevated to no more than 30° for 2 to 6 hours after the procedure. The amount of time depends on the type of closure device the provider uses. The client will receive a mild sedative for relaxation and comfort prior to the procedure. A soft knee brace can help keep the client from bending the knee after the procedure.

A nurse is administering subcutaneous heparin to a client who is at risk for deep vein thrombosis. Which of the following actions should the nurse take? a. Administer the medication into the client's abdomen b. Inject the medication into a muscle c. Massage the site after administering the medication d. Use a 22-gauge needle to administer the medication

A. Administer the medication into the client's abdomen---The heparin should be administered into the client's abdomen.

A nurse is teaching a client who has coronary artery disease about the difference between angina pectoris and myocardial infarction (MI). Which of the following manifestations should the nurse identify as indications of MI? (Select all that apply.) a. Nausea and vomiting b. Diaphoresis and dizziness c. Chest and left arm pain that subsides with rest d. Anxiety and feelings of doom e. Bounding pulse and bradypnea

A. Nausea and vomiting B. Diaphoresis and dizziness D. Anxiety and feelings of doom--- Nausea, vomiting, epigastric distress, diaphoresis (sweating), dizziness, fatigue, anxiety, and feelings of doom and fear are common manifestations of MI.

A nurse is caring for an older adult client who had an acute myocardial infarction (MI). When assessing this client, the nurse should identify that older adults are prone to complications of MI from poor tissue perfusion because of which of the following age-related factors? a. Peripheral vascular resistance increases. b. The sensitivity of blood pressure-adjusting baroreceptors increases. c. Blood is hypercoagulable and clots more quickly. d. Cardiac medications are less effective.

A. Peripheral vascular resistance increases.----Older adult clients are more prone to complications from poor tissue perfusion following an acute MI because peripheral vascular resistance increases with aging. This results from calcification and loss of elasticity of the blood vessels.

A nurse is planning to administer digoxin to a client who has heart failure. Which of the following laboratory results is the priority for the nurse to review prior to administering the medication? A.) Potassium B.) Hemoglobin C.) Creatinine D.) Blood urea nitrogen

A.) Digoxin is a cardiac glycoside medication used to improve myocardial contractility, increasing stroke volume and cardiac output in a client who has heart failure. During therapy, the nurse should closely monitor the client's potassium level as hypokalemia increases the risk of digitalis toxicity and cardiac arrhythmias.

A nurse is providing teaching about a heart healthy diet to a group of clients with hypertension. Which of the following statements by one of the clients indicates a need for further teaching? A.) "I may eat 10 ounces of lean protein each day." B.) "Fresh fruits make a good snack option." C.) "I will replace table salt with dried herbs." D.) "I may thicken gravies with cornstarch as I cook."

A.) Lean meats should be limited to 5 to 6 oz per day. This statement by a client requires additional teaching.

A nurse is teaching about necessary baseline examinations with a female client who is to start taking atorvastatin. Which of the following baseline examinations should the nurse include in the teaching? A.) Liver function tests B.) Hearing test C.) Papanicolaou test D.) Dental examination

A.) The nurse should inform the client that statins such as atorvastatin can cause liver damage and should not be taken by clients who have a history of liver disease. The client should undergo baseline liver function testing before beginning therapy, and every 6 to 12 months thereafter.

When checking a client's capillary refill, the nurse finds that the color returns in 10 seconds. The nurse should understand that this finding indicates which of the following? A.) Arterial insufficiency B.) Venous insufficiency C.) Within the expected range D.) Thrombus formation in the vein

A.) To test capillary refill, a nurse presses on the client's nail beds to produce blanching and then measures the time it takes for the color to return. With adequate arterial capillary perfusion, the color should return within 3 seconds. If the skin color takes longer than 3 seconds to return to normal, this indicates impaired arterial blood flow to the extremity.

A nurse is assessing an older adult client who is receiving digoxin. The nurse should recognize that which of the following findings is a manifestation of digoxin toxicity? a. anorexia b. ataxia c. photosensitivity d. jaundice

Anorexia

The nurse assesses a client who has just returned to the recovery area after undergoing coronary arteriography. Which information is of most concern? 1.Blood pressure is 154/78 mm Hg. 2.Pedal pulses are palpable at + 1. 3.Left groin has a 3-cm bruised area. 4.Apical pulse is 122 beats/min and regular.

Apical pulse is 122 beats/min and regular.

A nurse is teaching a client who has a new prescription for transdermal nitroglycerin to treat angina pectoris. Which of the following instructions should the nurse include in the teaching? A. Apply a new transdermal patch once a week. B. Apply the transdermal patch in the morning. C. Apply the transdermal patch in the same location as the previous patch. D. Apply a new transdermal patch when chest pain is experienced.

Apply the transdermal patch in the morning

Erectile dysfunction drugs such as sildenafil (Viagra) are contraindicated in clients taking nitrates for angina. What is the primary concern with concurrent administration of these drugs? a. They contain nitrates, resulting in an overdose. b. They decrease blood pressure and may result in prolonged and severe hypotension when combined with nitrates. c. They will adequately treat the patient's angina as well as erectile dysfunction. d. They will increase the possibility of nitrate tolerance developing and should be avoided unless other drugs can be used.

B) They decrease blood pressure and may result in prolonged and severe hypotension when combined with nitrates. Erectile dysfunction drugs such as sildenafil (Viagra), vardenafil (Levitra), and tadalafil (Cialis) decrease BP. When combined with nitrates, severe and prolonged hypotension may result.

A nurse is teaching a client about the proper placement of a nitroglycerin patch. Which of the following statements by the client indicates an understanding of the teaching? a. "I'll apply the patch over areas of my body with little fatty tissue." b. "I can place the patch on any area of my body without hair." c. "I'll put the patch on the same site as the previous patch." d. "I have to apply the patch directly over my heart."

B. "I can place the patch on any area of my body without hair."--- The nitroglycerin transdermal patch should be applied to skin that is free from hair because hair creates a physical barrier to absorption.

A nurse is providing discharge teaching for a client who had a left total hip arthroplasty. Which of the following client statements indicates the teaching was effective? a. "I should expect swelling of the affected leg for several weeks." b. "I should not cross my legs at the ankles or knees." c. "I will inspect my hip incision every other day for redness." d. "I can bend over at the hip to pick up objects."

B. "I should not cross my legs at the ankles or knees."---The nurse should instruct the client to avoid crossing the legs at the knees or ankles because this can result in the dislocation of the femoral head

A nurse is teaching a client who has a new prescription for hydrochlorothiazide for management of hypertension. Which of the following instructions should the nurse include? a. "Take this med before bedtime" b. "Monitor for leg cramps" c. "Avoid grapefruit juice" d. "Reduce intake of potassium-rich foods"

B. "Monitor for leg cramps"--- Hydrochlorothiazide can cause hypokalemia. The client should monitor for manifestations of hypokalemia, such as fatigue,tachycardia, leg cramps, & muscle weakness

A nurse is preparing to administer nitroglycerin topical ointment to a client who has angina. Which of the following actions should the nurse take? a. Cover the applied ointment with cotton gauze b. Apply the ointment using a dose-measuring applicator c. Apply the ointment using the index finger d. Massage the ointment into the client's skin

B. Apply the ointment using a dose-measuring applicator---The nurse should apply the ointment using a dose-measuring applicator. This allows the nurse to measure the correct dose the client is to receive.

A nurse is assessing a client who is experiencing chest pain. Which of the following medications should the nurse expect to administer to suppress the aggregation of platelets? a. Nitroglycerin b. Aspirin c. Morphine d.Metoprolol

B. Aspirin---Aspirin suppresses platelet aggregation, producing an immediate antithrombotic effect. The client should chew the first dose of aspirin to allow rapid absorption.

A nurse is caring for a client who has hypertension and has a potassium level of 6.8 mEw/L. Which of the following actions should the nurse take? a. Suggest that the client use a salt substitute b. Obtain a 12-lead ECG c. Advise the client to add citrus juices and bananas to her diet d. Obtain a blood sample for a serum sodium level

B. Obtain a 12-lead ECG--- The pt is at risk for dysrhythmias as well as cardiac arrest. The nurse should obtain a 12-lead ECG to monitor for cardiac changes

A nurse is establishing health promotion goals for a female client who smokes cigarettes. Has hypertension, and has a BMI of 26. Which of the following goals should the nurse include? a. The client will list foods that are high in calcium, which should be avoided b. The client will walk for 30 min/5 days a week c. The client will increase calorie intake by 200 cal per day d. The client will replace cigarettes with smokeless tobacco products

B. The client will walk for 30 min/5 days a week--- CDC recommends include engaging in a moderate exercise, such as walking, for a total of 150 min each week.

A client is teaching a client who has a new prescription for hydrochlorothiazide for management of hypertension. Which of the following instructions should the nurse include? A.) "Take this medication before bedtime." B.) "Monitor for leg cramps." C.) "Avoid grapefruit juice.' D.) "Reduce intake of potassium-rich foods."

B.) Hydrochlorothiazide can cause hypokalemia. The client should monitor for manifestations of hypokalemia, such as fatigue, tachycardia, leg cramps, and muscle weakness.

A nurse is assessing a client who has hypercholesterolemia and is receiving simvastatin. Which of the following findings should the nurse recognize as a potential adverse effect? A.) Urinary retention B.) Muscle weakness C.) Orthostatic hypotension D.) Blurred vision

B.) Muscle weakness

A nurse is caring for a client who has thrombophlebitis and is receiving heparin by continuous IV infusion. The client asks the nurse how long it will take for the heparin to dissolve the clot. Which of the following responses should the nurse give? A) "It usually takes heparin at least 2 to 3 days to reach a therapeutic blood level." B) "A pharmacist is the person to answer that question." C) "Heparin does not dissolve clots. It stops new clots from forming." D) "The oral medication you will take after this IV will dissolve the clot."

C) "Heparin does not dissolve clots. It stops new clots from forming."

A nurse is reviewing the laboratory values of a client who had a MI 3 hr ago. The nurse should expect which of the following lab values to be elevated? a. Aspartate aminotransferase (AST) b. Unconjugated bilirubin c. Troponin I d. Serum amylase

C) Troponin I--- Cardiac troponin I & cardiac troponin T are biochemical markers that are specific to myocardial cell injury. A client who has myocardial cell damage can have elevated troponin levels within 2-3 hr. Cardiac troponin I levels can peak in 10-24 hr & stay elevated for 7-10 days. Cardiac troponin T levels can peak within 10-24 hr stay elevated for 10-14 days.

A home health nurse is performing an assessment on a client who is 1 week postoperative following a total knee replacement. Which of the following statements by the client indicates an understanding of the teaching? a. "I will discontinue the blood thinner my doctor prescribed once I am at home." b. "I will keep a pillow under my knee when I am in bed." c. "I plan to use a walker to help me get around." d. "I will discontinue using the CPM machine when I get home."

C. "I plan to use a walker to help me get around."--- The nurse should identify that the client will receive a prescription for a walker, cane, or crutches to promote ambulation following a total knee replacement.

A nurse in an urgent care clinicis obtaining a history from aclient who has type 2 diabetes mellitus and a recent diagnosis of hypertension. This is the second time in 2 weeks that the client experienced hypoglycemia. Which of the following client data should the nurse report to the provider? A. Takes psyllium daily as a fiber laxative B. Drinks skim milk daily as a bedtime snack C. Takes metoprolol daily after meals D. Drinks grapefruit juice daily with breakfast

C. Takes metoprolol daily after meals

A nurse assessing a client determines that he is in the compensatory stage of shock. Which of the following findings support this conclusion? 1. confusion 2. lethargy 3. unconsciousness 4. petechiae

Confusion; lethargy is the progressive stage of shock, unconsciousness is the irreversible stage of shock, and petechiae is the progressive stage of shock.

A nurse in an outpatient clinic is assessing a middle adult client as part of a routine physical examination. The client's BP is 142/88. BMI is 31. Current smoker. The nurse should identify that this client has multiple risk factors for which disorder? a.Testicular cancer b.Cardiovascular disease c.Depression d.Thyroid disease

Correct: b. Cardiovascular disease Response Feedback:Risk factors for cardiovascular disease include BP elevation, obesity, smoking, and a sedentary lifestyle.

A nurse is instructing a client who has a new diagnosis of Raynaud's disease about preventing the onset of manifestations. Which of the following client statements should indicate to the nurse the need for additional teaching? A) "I will wear gloves when removing food from the freezer." B) "I will try to anticipate and avoid stressful situations when possible." C) "I will complete the smoking cessation program I started." D) "I will take my medications at the first time of an attack."

D) "I will take my medications at the first time of an attack."

A nurse is caring for a client who returns to the nursing unit from the recovery room after a sigmoid colon resection for adenocarcinoma. The client had an episode of intraoperative bleeding. Which finding indicates to the nurse that the client may be developing hypovolemic shock? A) Decrease in the respiratory rate from 20 to 16/min. B) Decrease in the urinary output from 50mL to 30mL per hour. C) Increase in the temperature from 37.5C (99.5F) to 38.6C (101.5F). D) Increase the heart rate from 88 to 110/min.

D) Increase the heart rate from 88 to 110/min

A nurse is teaching about risk factors of developing a stroke with a group of older adult clients. Which of the following nonmodifiable risk factors should the nurse include in the teaching? A) History of smoking B) Obesity C) History of hypertension D) Race

D) Race

A nurse is preparing to administer Dabigatran to a client who has A Fib. The nurse should explain that the purpose of this medication is which of the following? a. to convert atrial fibrillation to sinus rhythm b. to dissolve clots in the bloodstream c. to slow the response of the ventricles to the fast atrial impulses d. to reduce the risk of stroke in clients who have atrial fibrillation.

D. Clients who have atrial fibrillation are at an increased risk for thrombus formation and subsequent embolization to the brain. Anticoagulants, such as dabigatran, help prevent thrombus formation.

A nurse is providing teaching to a client who has stable angina and a new prescription for nitroglycerin oral, sustained-release capsules. Which of the following instructions should the nurse include? A.) Take 1 capsule at the onset of anginal pain. B.) Take 1 capsule at the onset of anginal pain. C.) Take the medication with meals. D.) Swallow the capsules whole.

D.) Do not crush or chew either

A nurse is assessing for paradoxical blood pressure on a client who has constrictive pericarditis. Which of the following findings should the nurse expect? Apical pulse rate is different than the radial pulse rate Decrease in systolic pressure by more than 10 mm Hg during inspiration Increase in heart rate by 20% when moving from sitting to standing Drop in systolic BP by 20 mm Hg when changing positions

Decrease in systolic pressure by more than 10 mm Hg during inspiration The nurse should expect a client who has constrictive pericarditis to have a decrease in systolic pressure by more than 10 mm Hg during inspiration, which is paradoxical blood pressure. This is also an expected finding for a client who has pulmonary hypertension or cardiac tamponade.

A nurse is providing teaching to a client who has hypertension and a new prescription for captopril. Which of the following instructions should the nurse provide? Do not use salt substitutes while taking this medication. Take the medication with food. Count your pulse rate before taking the medication. Expect to gain weight while taking this medication.

Do not use salt substitutes while taking this medication. Captopril, an ACE inhibitor, can cause hyperkalemia due to potassium retention by the kidney. The client should avoid salt substitutes, as most of them are high in potassium.

A nurse in a cardiac care unit is caring for a client with acute right-sided heart failure. Which of the following findings should the nurse expect? Decreased brain natriuretic peptide (BNP). Elevated central venous pressure (CVP). Increased pulmonary artery wedge pressure (PAWP). Decreased specific gravity

Elevated central venous pressure (CVP). CVP is a measurement of the pressure in the right atria or ventricle at the end of diastole. An elevated CVP is indicative of heart failure.

A nurse is caring for a client who is taking digoxin for heart failure and develops indications of severe digoxin toxicity. Which of the following medications should the nurse prepare to administer? a. fab antibody fragments b. flumazenil c. acetylcysteine d. naloxone

Fab antibody fragments

A nurse is reinforcing teaching with an older adult client who has just undergone insertion of a permanent pacemaker. The nurse should emphasize that a sign of pacemaker malfunction the client should report to the provider is 1. increased urine output 2. rapid pulse 3. fatigue 4. sneezing

Fatigue; pacemaker malfunction causes bradycardia and a drop in cardiac output. This can cause hypoxia with manifestations of weakness, fatigue, and dizziness.

A nurse is providing teaching to a client who has hypertension and a new prescription for verapamil. Which of the following beverages should the nurse tell the client to avoid while taking this medication? A. Milk B. Orange juice C. Coffee D. Grapefruit juice

Grapefruit Juice

The nurse is caring for a client who has just had a cardiac catheterization. The post procedure nursing care plan for this client should include which of the following nursing interventions? 1. Have the client rest in bed for 2-6 hours 2. keep the involved leg slightly flexed 3. elevate the head of the ed 45 degrees 4. keep the client NPO for 4 hours

Have the client rest in bed for 2-6 hours.

A nurse is caring for a client who has thrombophlebitis and is receiving Heparin by continuous IV infusion. The client asks the nurse how long it will take for the Heparin to dissolve the clot. Which of the following responses should the nurse give? a. It usually takes heparin at least 2 to 3 days to reach a therapeutic blood level. b. A pharmacist is the person to answer that question. c. Heparin does not dissolve clots. It stops new clots from forming. d. The oral medication you will take after this IV will dissolve the clot.

Heparin does not dissolve clots. It stops new clots from forming

A client is preparing for an exercise stress test. Which of the following comments should indicate to the nurse that the client requires further reinforcement of teaching? 1. I will not smoke prior to my test 2. I'll take my heart medications the morning of my test 3. I'll get 8 hours of sleep the bight before the test 4. I'll skip my coffee the morning of my test

I'll take my heart medications the morning of my test; client should avoid medications that will prevent fluctuations in heart rate during the test

A nurse is assessing a client who is receiving a continuous IV infusion of dopamine. Which of the following findings should the nurse recognize as a therapeutic effect? A. Increased pulse B. Increased urine output C. Decreased blood pressure D. Decreased dysrhythmias

Increased urine output

A client tells the nurse that he is concerned because his provider told him he has a heart murmur. The nurse should explain to the client that a murmur. A) is a high-pitched sound due to a narrow valve. B) is an extra sound due to blood entering an inflexible chamber. C) means that there is some inflammation around your heart. D) indicates turbulent blood flow through a valve.

Indicates turbulent blood flow through a valve

A nurse on a telemtry unit is caring for a client who has premature ventricular contractions. While sitting in a chair, the client reports feeling lightheaded. If the client is having PVCs, which of the following findings should the nurse expect when auscultating the client's apical pulse? 1. bounding pulsations 2. irregular pulsations 3. tachycardia 4. bradycardia

Irregular pulsations; the pause in usual heart rhythm results in irregular apical pulse

A nurse is preparing a client for an echocardiogram the following day. Which of the following instructions should the nurse include about the test? 1. it might cause slight discomfort in the chest area 2. it takes about 5-10 minutes 3. it requires lying quietly on one side 4. it is best to have no food or beverages the day of the test

It requires lying quietly on one side; an echocardiogram takes 30-60 minutes and is painless

A nurse is reviewing the laboratory results of a male adult client who is at risk for peripheral arterial disease from atherosclerosis. The nurse should identify that which of the following results places the client at risk? Triglycerides 130 mg/dl Blood glucose 92 mg/dl LDL 172 mg/dl HDL 84 mg/dl

LDL 172 mg/dL The nurse should identify that an LDL of 172 mg/dL places the client at risk for peripheral arterial disease from atherosclerosis. The expected reference range for an adult is less than 130 mg/dL.

A nurse is assessing a client who has hypercholestterolemia and is receiving simvastatin. Which of the following findings should the nurse recognize as a potential adverse effect? Urinary retention Muscle weakness Orthostatic hypotension Blurred vision

Muscle weakness Myopathy is an adverse effect of this medication. Signs of myopathy include muscle aches, tenderness, and muscle weakness.

A nurse is caring for a client who enters the emergency department complaining of severe chest pain. Which of the following interventions should the nurse implement to determine if the client is experiencing a myocardial infarction? 1. check BP 2. auscultate heart tones 3. perform 12-lead ECG 4. determine if pain radiates to left arm

Perform 12-lead ECG

A nurse is reading a client's ECG tracing. Which component of the ECG should the nurse examine to determine the time it takes for ventricular depolarization and repolarization? 1. PR interval 2 QT interval 3. ST segment 4. QRS complex

QT interval; reflects the time it takes for ventricular depolarization and repolarization.

A nurse is reviewing the health history for a client who has angina pectoris and a prescription for propranolol hydrochloride PO 40 mg twice daily. Which of the following findings in the history should the nurse report to the provider? a. hypothyroidism b. bronchial asthma c. htn d. migraine headaches

The client has a history of bronchial asthma.

A nurse is establishing health promotion goals for a female client who smokes cigarettes, has HTN, and has a BMI of 26. Which of the following goals should the nurse include? The client will list foods that are high in calcium, which should be avoided. The client will walk for 30 min 5 days a week. The client will increase calorie intake by 200 cal per day. The client will replace cigarettes with smokeless tobacco products.

The client will walk for 30 min 5 days a week. CDC recommendations include engaging in a moderate exercise, such as walking, for a total of 150 min each week.

A nurse is caring for a client who has deep vein thrombosis and has been on Heparin continuous infusion for 5 days. the provider prescribes Warfarin without discontinuing the Heparin the client asks the nurse why both anticoagulants are necessary. which of the following statements should the nurse make? A. "Warfarin takes several days to work, so the IV heparin will be used until the warfarin reaches a therapeutic level." B. "I will call the provider to get a prescription for discontinuing the IV heparin today." C. "Both heparin and warfarin work together to dissolve the clots." D. "The IV heparin increases the effects of the warfarin and decreases the length of your hospital stay."

Warfarin take several days to work, so the IV Heparin will be used until the warfarin reaches a therapeutic level

A patient with aortic valve stenosis is being admitted for valve replacement surgery. Which assessment finding documented by the nurse is indicative of this condition? a. Systolic murmur b. Distended neck veins c. Splinter hemorrhages d. Pulse deficit

a. Systolic murmur The turbulent blood flow across a diseased valve results in a murmur, aortic stenosis produces a systolic murmur

Which one of these clinical assessment findings should the nurse document and report for a client who has a diagnosis of right sided heart failure indicating a complication with cardiac perfusion? a. peripheral edema, ascites, JVD b. weight gain, crackles, JVD c. periorbital edema, moist cough, ascites d. frothy, pink sputum, RR - 30, anxious

a. peripheral edema, ascites, JVD

A client who has a history of myocardial infarction (MI) is prescribed aspirin 325 mg. The nurse recognizes that the aspirin is given due to which of the following actions of the medications? A) analgesic B) anti-inflammatory C) antiplatelet aggregate D) antipyretic

antiplatelet aggregate

When evaluating a patient's knowledge regarding a low-sodium, low-fat cardiac diet, the nurse recognizes additional teaching is needed when the patient selects what food? a. Angel food cake b. Baked flounder c. Canned chicken noodle soup d. Baked potato with margarine

c. Canned chicken noodle soup canned soups are high in sodium content

Which patient is at GREATEST risk for sudden cardiac death (SCD)? a. A 42 yo white woman with HTN and dyslipidemia b. A 52 yo African American man with left ventricular failure c. A 62 yo obese man with diabetes mellitus and high cholesterol d. A 72 yo Native American woman with a family history of heart disease

b. A 52 yo African American man with left ventricular failure Patients with left ventricular failure after MI are at greatest risk for SCD. Other risk factors for SCD: male gender, African American race, family history of premature atherosclerosis, tobacco use, diabetes mellitus, hypercholesterolemia, hypertension, and cardiomyopathy

A nurse is caring for a client who has pericarditis and reports feeling a new onset of palpitations and shortness of breath. Which of the following assessments should indicate to the nurse that the client may have developed atrial fibrillation? a. Different blood pressures in the upper limbs b. Different apical and radial pulses c. Differences between oral and axillary temperatures d. Differences in upper and lower lung sounds

b. Different apical and radial pulses different apical and radial pulses indicate a pulse deficit, which is a classic sign of A-fib

A nurse is caring for a client who has infective endocarditis. Which of the following manifestations is the priority for the nurse to monitor for? a. Anorexia b. Dyspnea c. Fever d. Malaise

b. Dyspnea b would be the priority and checked 1st c would be checked 2nd

A patient with a history of myocardial infarction is scheduled for a transesophageal echocardiogram to visualize a suspected clot in the left atrium. What information should the nurse include when teaching the patient about this diagnostic study? a. Food and fluids are restricted for 2 hours before the procedure b. IV sedation may be administered to help the patient relax c. Ambulation is restricted for up to 6 hours before the procedure d. Contrast medium is injected into the esophagus to enhance images

b. IV sedation may be administered to help the patient relax Sedation eases the insertion of the tube into the esophagus. Food/fluids are restricted for at least 6 hours beforehand. Contrast medium is administered IV to evaluate the direction of blood flow if a septal defect is suspected.

A nurse is collecting a medication hx from a client who is scheduled to have a cardiac catheterization. Which of the following medications taken by the client interacts with contrast material and places the client at risk for acute kidney injury? a. Atorvastatin b. Metformin c. Nitroglycerin d. Carvedilol

b. Metformin Metformin interacts with contrast dye and can cause acute kidney damage.Atorvastatin is contraindicated for a client who has active hepatic disease, but it does not interact with contrast material.Nitroglycerin is contraindicated for a number of conditions including increased intracranial pressure, but it does not interact with contrast material.Carvedilol is contraindicated for a number of conditions including 2nd and 3rd degree heart block, but it does not interact with contrast material

In palpating the patient's pedal pulses, the nurse determines the pulses are absent. What factor could contribute to this result? a. Cardiac dysrhythmias b. Hyperthyroidism c. Atherosclerosis d. Arteriovenous fistula

c. Atherosclerosis Atherosclerosis can cause absent peripheral pulses. Feet would be cool/discolored.

A patient admitted to the emergency department 24 hours ago with complaints of chest pain was diagnosed with an ST-segment-elevation myocardial infarction (STEMI). What complication of myocardial infarction should the nurse anticipate? a. Unstable angina b. Sudden cardiac death c. Cardiac dysrhythmias d. Cardiac tamponade

c. Cardiac dysrhythmias present in 80-90% of patients after MI. Unstable angina is considered a precursor vs a complication. Tamponade is a rare event, and SCD would happen afterward

When looking at the EKG of the patient, the nurse knows that the QRS complex recorded on the EKG represents which part of the heart's beat? a. The length of time it takes for the impulse to travel from the atria to the ventricles b. Depolarization of the atria c. Depolarization from the atrioventricular (AV) node throughout the ventricles d. Repolarization of the ventricles

c. Depolarization from the atrioventricular (AV) node throughout the ventricles P wave represents depolarization of the atria QRS represents depolarization from the AV node throughout the ventricles T wave represents the repolarization of the ventricles

Which factor should be considered when caring for a women with suspected CAD? a. Women are more likely to develop collateral circulation b. Increased risk is present before menopause c. Fatigue may be the first symptom d. Classic signs and symptoms are expected

c. Fatigue may be the first symptom fatigue rather than pain or SOB, due to women not displaying normal signs of ischemia. Neck, throat, and back pain may be symptoms. Occurs more frequently after menopause. Men are more likely to develop collateral circulation

In caring for the patient with angina, the patient said, "While I was having a bowel movement, I started having the worst chest pain ever, like before I was admitted. I called for a nurse, then the pain went away." What further assessment data should the nurse obtain from the patient? a. What is your pain level on a scale of 0-10? b. What precipitated the pain? c. In what areas did you feel this pain? d. Has the pain changed this time?

c. In what areas did you feel this pain? Using PQRST, radiation of pain wasn't acknowledged

The nurse is providing care for a patient who has decreased cardiac output related to heart failure. What should the nurse recognize about cardiac output? a. It is determined by measuring the electrical activity of the heart and the patient's heart rate b. It is the patient's average resting heart rate multiplied by the patient's mean arterial blood pressure c. It is calculated by multiplying the patient's stroke volume by the heart rate d. It is the average amount of blood ejected during one complete cardiac cycle

c. It is calculated by multiplying the patient's stroke volume by the heart rate

The blood pressure of an older adult patient admitted with pneumonia is 160/70 mmHg. What is an age-related change that contributes to this finding? a. Increased parasympathetic activity b. Decreased adrenergic sensitivity c. Loss of elasticity in arterial vessels d. Stenosis of the heart valves

c. Loss of elasticity in arterial vessels Increasing resistance to flow, pressure is increased within the blood vessel, and hypertension results

A 74 yo man with a history of prostate cancer and hypertension is admitted to the emergency department with substernal chest pain. Which action will the nurse complete before administering sublingual nitroglycerin? a. Assess for CAD risk factors b. Administer morphine sulfate IV c. Obtain a 12-lead EKG d. Auscultate heart and lung soungs

c. Obtain a 12-lead EKG If someone is experiencing chest pain:1. administer supplemental oxygen and position patient in upright position2. assess vitals3. obtain 12-lead4. provide prompt pain relief first with nitrate followed by opioid analgesic if needed5. auscultate heart sounds

The nurse is examining the EKG of a patient just admitted with a suspected MI. Which of the following EKG changes is MOST indicative of prolonged or complete coronary occlusion? a. Sinus tachycardia b. Prolonged PR interval c. Pathologic Q wave d. Fibrillarory P waves

c. Pathologic Q wave Presence of this often accompanies STEMI, indicative of complete coronary occlusion


Kaugnay na mga set ng pag-aaral

Pregnancy, labor, childbirth, postpartum- uncomplicated

View Set

Unit 11 and 12 - Landlord & Tenant and Property Management

View Set

Jewish, Early Christian, and Byzantine Art

View Set

WEEK No. (9)- Ninth Lecture-2023- Selected Texts from Social Theories

View Set